Assessment Exam Flashcards

1
Q

Interoceptive conditioning is one of the components of the cognitive-behavioral treatment of Panic Disorder and may include which of the following strategies?
Select one:

A.
yelling stop whenever an undesirable thought occurs

B.
maintaining a panic diary

C.
focusing on a pleasant experience or fantasy

D.
breathing through a thin straw

A

Answer D is correct: Interoceptive conditioning has been identified as an effective component of cognitive-behavioral therapy for Panic Disorder and involves having the individual engage in activities that produce physical sensations associated with a panic attack such as breathing through a thin straw, spinning in a swivel chair, and running in place.

Answers A, B, and C: See explanation for answer D.
The correct answer is: breathing through a thin straw

How well did you know this?
1
Not at all
2
3
4
5
Perfectly
2
Q

A better prognosis for Schizophrenia is associated with all of the following except:

Select one:

a.
Female gender

b.
An early onset of symptoms

c.
A family history of a mood disorder

d.
The presence of a precipitating event

A

Answer B is correct: A late (not early) onset of symptoms is associated with a better prognosis.

Answers A, C, and D are incorrect: Female gender, a family history of a mood disorder, and the presence of a precipitating event have been linked to a better prognosis.

The correct answer is: An early onset of symptoms

How well did you know this?
1
Not at all
2
3
4
5
Perfectly
3
Q

The assumption underlying dialectical behavior therapy (Linehan, 1993) as a treatment for Borderline Personality Disorder is that this disorder is caused by:
Select one:

A.
pervasive emotion dysregulation.

B.
dysfunctional object relations.

C.
coercive family interactions.

D.
a lack of response-contingent reinforcement.

A

Answer A is correct: Dialectical behavior therapy (DBT) was originally developed by Linehan (1993) as a treatment for Borderline Personality Disorder but has since been applied to several other disorders including eating disorders, ADHD in adults, and depression in older adults. According to Linehan, Borderline Personality Disorder is the result of a combination of emotion dysregulation, emotional vulnerability, and an invalidating environment.

Answer B: DBT relies primarily on cognitive-behavioral principles and does not consider Borderline Personality Disorder to be the result of dysfunctional object relations.
Answer C: Coercive family interactions have been linked to aggressive behavior in children by Patterson (1992).
Answer D: Some behavioral models of depression describe it as the result of a lack of response-contingent reinforcement.

The correct answer is: pervasive emotion dysregulation.

How well did you know this?
1
Not at all
2
3
4
5
Perfectly
4
Q

The revised learned helplessness model proposed by Abramson, Metalsky, and Alloy (1989) identifies ___________ as the key contributor to depression.
Select one:

A.
a high rate of self-punishment

B.
emotional oversensitivity

C.
the depressive cognitive triad

D.
hopelessness

A

Answer D is correct: According to Abramson, Metalsky, and Alloy’s (1989) version of the learned helplessness model, a sense of hopelessness is a proximal cause of some types of depression. From this perspective, the attributions a person makes about negative events in his/her life contribute to depression only to the degree that they foster a sense of hopelessness.

Answer A: A high rate of self-punishment has been identified by Rehm (1987) as a contributor to depression.
Answer B: Emotional oversensitivity is not a component of the learned helplessness model of depression.
Answer C: Beck’s (1976) cognitive theory views depression as involving a “depressive cognitive triad” that consists of negative, illogical self-statements about oneself, the world, and the future.

The correct answer is: hopelessness

How well did you know this?
1
Not at all
2
3
4
5
Perfectly
5
Q

A DSM diagnosis of Antisocial Personality Disorder requires a history of symptoms of Conduct Disorder prior to ___ years of age.
Select one:

A.
7

B.
10

C.
13

D.
15

A

Answer D is correct: As defined in the DSM, a diagnosis of Antisocial Personality Disorder requires that the person be at least 18 years of age, that there is evidence of Conduct Disorder prior to age 15, and that the person has exhibited at least three characteristic symptoms involving the violation of the rights of others since the age of 15.

Answers A, B, and C: See explanation for answer D.

The correct answer is: 15

How well did you know this?
1
Not at all
2
3
4
5
Perfectly
6
Q

Dalal D., age 21, displays several active psychotic symptoms including persecutory delusions, auditory hallucinations, incoherence, and loosening of associations. Assuming that her symptoms started suddenly three months ago and that she has no previous history of similar symptoms, your tentative diagnosis would be which of the following?
Select one:

A.
Schizophrenia

B.
Brief Psychotic Disorder

C.
Schizoaffective Disorder

D.
Schizophreniform Disorder

A

Answer D is correct: The disorders listed in the answers to this question share several characteristics but also differ in terms of duration and/or certain key symptoms. Dalal’s symptoms and their duration are most suggestive of Schizophreniform Disorder.
Answer A: Although Dalal’s symptoms suggest Schizophrenia, they do not meet the DSM requirement of a duration of least 6 months for a diagnosis of this disorder.
Answer B: The duration of Brief Psychotic Disorder is from one day to one month.
Answer C: For a diagnosis of Schizoaffective Disorder, mood symptoms must co-occur with psychotic symptoms except during a period of at least two weeks in which delusions or hallucinations are present without prominent mood symptoms.

The correct answer is: Schizophreniform Disorder

How well did you know this?
1
Not at all
2
3
4
5
Perfectly
7
Q

The prognosis for a child with autism is best if the child:
Select one:

A.
does not have delays in motor development.

B.
displays some ability to communicate verbally by age five or six.

C.
has one or more savant abilities.

D.
does not have a family history of a mental disorder.

A

Answer B is correct: Most individuals with autism continue to have significant impairments throughout their lives. However, a better prognosis has been linked to the ability to communicate verbally by age five or six, an IQ of 70 or above, and a later onset of symptoms.

Answers A, C, and D: See explanation for answer B.

The correct answer is: displays some ability to communicate verbally by age five or six.

How well did you know this?
1
Not at all
2
3
4
5
Perfectly
8
Q

Sensate focus is based on the assumption that sexual dysfunction is often due to:
Select one:

A.
mistaken beliefs about sexuality.

B.
decreased libido.

C.
intrapsychic conflicts related to sexuality.

D.
performance anxiety.

A

Answer D is correct: Sensate focus was developed by Masters and Johnson (1970) and consists of a series of exercises designed to help a couple overcome anxiety related to intimacy and sexual intercourse by focusing on pleasurable sensations. It has been found useful for treating premature ejaculation and other sexual dysfunctions that are due to performance anxiety.

Answer A: Masters and Johnson recognized the impact of misinformation and mistaken beliefs on sexual functioning, but these are not the targets of sensate focus.
Answer B: Decreased libido could, of course, be caused by performance anxiety; but sensate focus is not based on the assumption that sexual dysfunction is often due to decreased libido.
Answer C: Masters and Johnson did not view sexual dysfunction as a result of intrapsychic conflicts.

The correct answer is: performance anxiety.

How well did you know this?
1
Not at all
2
3
4
5
Perfectly
9
Q

For children and adolescents, a diagnosis of Cyclothymic Disorder requires the presence of symptoms for at least:
Select one:

A.
six months.

B.
twelve months.

C.
twenty-four months.

D.
thirty-six months.

A

Answer B is correct: Cyclothymic Disorder is characterized by a disturbance of mood involving hypomanic episodes and periods of depressed mood for an extended period of time. A DSM diagnosis of Cyclothymic Disorder requires the presence of symptoms for at least two years in adults and one year in children and adolescents.
Answers A, C, and D: See explanation for answer B.

The correct answer is: twelve months.

How well did you know this?
1
Not at all
2
3
4
5
Perfectly
10
Q

A DSM diagnosis of Bipolar II disorder requires which of the following?
Select one:

A.
one or more manic episodes

B.
one or more mixed episodes

C.
manic and major depressive episodes

D.
hypomanic and major depressive episodes

A

Answer D is correct: A diagnosis of Bipolar II disorder requires at least one major depressive episode and at least one hypomanic episode.
Answers A, B, and C: See explanation for Answer D.

The correct answer is: hypomanic and major depressive episodes

How well did you know this?
1
Not at all
2
3
4
5
Perfectly
11
Q

As defined by the DSM, a diagnosis of Attention-Deficit/Hyperactivity Disorder (ADHD) requires which of the following?
Select one:

A.
an onset of symptoms prior to 4 years of age

B.
a duration of symptoms of at least 6 months

C.
markedly impaired academic performance

D.
negativistic, hostile, and defiant behavior

A

Answer B is correct: Of the diagnostic criteria listed in the answers, a duration of symptoms of at least 6 months is the only one required by the DSM-5 for a diagnosis of ADHD.

The correct answer is: a duration of symptoms of at least 6 months

How well did you know this?
1
Not at all
2
3
4
5
Perfectly
12
Q

According to the catecholamine hypothesis:
Select one:

A.
mania is due to a deficiency in norepinephrine.

B.
depression is due to a deficiency in norepinephrine.

C.
mania is due to excessive acetylcholine.

D.
depression is due to excessive acetylcholine.

A

Answer B is correct: According to the catecholamine hypothesis, at least some types of depression are due to a lower-than-normal level of norepinephrine.

Answer A: The catecholamine hypothesis predicts that mania is due to excessive norepinephrine.

Answers C and D: See explanation for answer B.

The correct answer is: depression is due to a deficiency in norepinephrine.

How well did you know this?
1
Not at all
2
3
4
5
Perfectly
13
Q

The prognosis for Schizophrenia has been linked to several factors. Which of the following is NOT associated with a better prognosis?
Select one:

A.
an acute and late onset of the disorder

B.
the presence of a precipitating event

C.
a family history of a mood disorder

D.
male gender

A

Answer D is correct: The course of Schizophrenia varies from individual to individual, but the disorder is usually chronic and complete remission is rare. However, research has identified several factors that are associated with a better prognosis including female gender, an acute and late onset of symptoms, the presence of a precipitating event, and a family history of a mood disorder.

Answers A, B, and C: See explanation for answer D.

The correct answer is: male gender

How well did you know this?
1
Not at all
2
3
4
5
Perfectly
14
Q

The most effective treatment for Obsessive-Compulsive Disorder is:
Select one:

A.
exposure therapy with response prevention.

B.
stress management with relaxation training.

C.
covert sensitization.

D.
stress inoculation training.

A

Answer A is correct: Exposing the individual to anxiety-arousing stimuli while prohibiting him or her from engaging in usual anxiety-reducing responses (rituals) is considered to be treatment-of-choice for OCD.
Answers B, C, and D: See explanation for response A.

The correct answer is: exposure therapy with response prevention.

How well did you know this?
1
Not at all
2
3
4
5
Perfectly
15
Q

Following cessation of alcohol use that has been prolonged and heavy, a 45-year old woman exhibits a number of symptoms including autonomic hyperactivity, hand tremor, and nausea. A diagnosis of Alcohol Withdrawal Delirium would be a more appropriate diagnosis than Alcohol Withdrawal if the woman also exhibits:
Select one:

A.
psychomotor agitation.

B.
tonic-clonic seizures.

C.
impaired attention and awareness.

D.
tardive dyskinesia.

A

Answer C is correct: The key to identifying the correct answer to this question is to know that disturbances in attention and awareness are essential features of Delirium.

Answer A: Psychomotor agitation is characteristic of Alcohol Withdrawal.
Answer B: In extreme cases, Alcohol Withdrawal can involve tonic-clonic seizures.
Answer D: Tardive dyskinesia is not a symptom of Delirium.

The correct answer is: impaired attention and awareness.

How well did you know this?
1
Not at all
2
3
4
5
Perfectly
16
Q

Marlatt and Gordon (1985) describe which of the following as the result of an “overlearned habit pattern”?
Select one:

A.
parasuicidal behavior

B.
substance addiction

C.
hypochondriasis

D.
paraphilias

A

Answer B is correct: Marlatt and Gordon (1985) describe addictions as the result of problematic learned habits or behaviors that have been acquired because they produce immediate gratification (reinforcement).
Answers A, C, and D: See explanation for answer B.

The correct answer is: substance addiction

How well did you know this?
1
Not at all
2
3
4
5
Perfectly
17
Q

Children with Tourette’s Disorder frequently exhibit significant problems in learning. This is most likely attributable to:
Select one:

A.
language disabilities.

B.
attention deficits and hyperactivity.

C.
lower-than-average IQ.

D.
peer and other social problems.

A

Answer B is correct: Hyperactivity, distractibility, and impulsivity are fairly common in individuals with Tourette’s Disorder and have been identified as a cause of school problems.

Answer A: Language disabilities are not associated with Tourette’s Disorder.

Answer C: A lower-than-average IQ is not characteristic of Tourette’s Disorder.

Answer D: Although impairments in social functioning are common, they have not been identified as a cause of academic failures.

The correct answer is: attention deficits and hyperactivity.

How well did you know this?
1
Not at all
2
3
4
5
Perfectly
18
Q

A DSM-5 diagnosis of Specific Learning Disorder requires that the individual’s academic skills be substantially below those expected for his or her:
Select one:

A.
chronological age.

B.
mental age.

C.
measured intelligence.

D.
academic aptitude.

A

Answer A is correct: A DSM-5 diagnosis of Specific Learning Disorder requires the individual’s academic skills to be “substantially and quantifiably below those expected for the individual’s chronological age” (APA, 2013, p. 67).

The correct answer is: chronological age.

How well did you know this?
1
Not at all
2
3
4
5
Perfectly
19
Q

As described in the DSM-5, a manic episode involves a period of abnormally and persistently elevated, expansive, or irritable mood plus:
Select one:

A.
inflated self-esteem or grandiosity.

B.
persistently increased activity or energy.

C.
involvement in activities that have a high potential for negative consequences.

D.
anxious distress.

A

Answer B is correct: Criterion A for both manic and hypomanic episodes in the DSM-5 includes “an emphasis on changes in activity and energy as well as mood” (2013, p. 4). Therefore, this is the best answer of those given.

Answers A and C: Inflated self-esteem and grandiosity and persistent involvement in activities that have a high potential for painful consequences are possible symptoms of a manic episode but are not required.

Answer D: Anxious distress is a possible accompanying symptom of Bipolar I Disorder and “with anxious distress” is included in the DSM-5 as a specifier for this disorder.

The correct answer is: persistently increased activity or energy.

How well did you know this?
1
Not at all
2
3
4
5
Perfectly
20
Q

Ethel E., age 36, has just been offered a promotion at work. Although she wants the increase in salary and prestige of the new position, she is planning to turn down the job because it will require extensive speaking in front of large groups of employees. Ethel has avoided situations that require her to speak to groups for as long as she can remember because public speaking is a very embarrassing experience for her. Whenever she has to speak in front of others, she is extremely anxious, her heart races and her palms become sweaty, and she becomes preoccupied with the concern that she will forget what she has planned to say. Ethel’s symptoms are most suggestive of which of the following disorders?
Select one:

A.
Panic Disorder

B.
Social Anxiety Disorder

C.
Avoidant Personality Disorder

D.
Obsessive-Compulsive Personality Disorder

A

Answer B is correct: Of the disorders listed, Ethel’s symptoms come closest to those required for a DSM diagnosis of Social Anxiety Disorder (Social Phobia). She has an intense fear of a specific social situation (public speaking) because it exposes her to scrutiny by others. Consequently, she avoids the situation.

Answer A: The diagnosis of Panic Disorder requires recurrent unexpected panic attacks. Although Ethel’s symptoms are consistent with panic attacks, they are linked to a specific situation and are not unexpected.

Answers C and D: Ethel’s symptoms are not characteristic of Avoidant Personality Disorder or Obsessive-Compulsive Personality Disorder.

The correct answer is: Social Anxiety Disorder

How well did you know this?
1
Not at all
2
3
4
5
Perfectly
21
Q

In prepubertal children, boys and girls are about equally affected by Major Depressive Disorder. In adolescents and adults:
Select one:

A.
the rates for males and females remain about equal.

B.
the rate for females is about 1.5 to 3 times the rate for males.

C.
the rate for females is about 4 to 5 times the rate for males.

D.
the rate for males is 2 to 3 times the rate for females.

A

Answer B is correct: According to the DSM-5, beginning in early adolescence, the rate of Major Depressive Disorder for females is 1.5 to 3 times the rate for males.

Answers A, C, and D: See explanation for answer B.

The correct answer is: the rate for females is about 1.5 to 3 times the rate for males.

How well did you know this?
1
Not at all
2
3
4
5
Perfectly
22
Q

A diagnosis of Disruptive Mood Dysregulation Disorder should not be assigned for the first time before the individual is _____ years of age or older than _____ years of age.
Select one:

A.
three; fifteen

B.
five; sixteen

C.
six; eighteen

D.
ten; eighteen

A

Answer C is correct. As described in the DSM-5, the diagnosis of Disruptive Mood Dysregulation Disorder must not be assigned for the first time when the individual is less than six years of age or more than 18 years of age, and the onset of symptoms must be prior to age 10.

Answers A, B, and D: See explanation for answer C.

The correct answer is: six; eighteen

How well did you know this?
1
Not at all
2
3
4
5
Perfectly
23
Q

Research has linked Obsessive-Compulsive Disorder to overactivity in which of the following?
Select one:

A.
caudate nucleus

B.
hippocampus

C.
suprachiasmatic nucleus

D.
medulla oblongata

A

Answer A is correct: Research using brain imaging techniques has found that the caudate nucleus (which is involved in the initiation and control of movement) tends to be overactive in individuals with OCD. The studies have also found that the reduction of obsessions and compulsions following treatment with cognitive-behavioral therapy or an SSRI is accompanied by a decrease in activity in the caudate nucleus.

Answers B, C, and D: See explanation for answer A.

The correct answer is: caudate nucleus

How well did you know this?
1
Not at all
2
3
4
5
Perfectly
24
Q

A DSM-5 diagnosis of Somatic Symptom Disorder requires the presence of one or more somatic symptoms that are distressing or cause significant disruption in daily life plus:
Select one:

A.
evidence that symptoms are not associated with a known medical condition.

B.
performance of excessive health-related behaviors or maladaptive avoidance of medical care.

C.
evidence that symptoms are not being feigned or voluntarily produced.

D.
excessive thoughts, feelings, or behaviors related to the symptoms.

A

Answer D is correct: Criterion B for the DSM-5 diagnosis of Somatic Symptom Disorder requires the presence of “excessive thoughts, feelings, or behaviors related to the somatic symptoms or associated health concerns” (APA, 2013, p. 311).

Answer A. For the diagnosis of Somatic Symptom Disorder, symptoms may or may not be associated with a medical condition.

Answer B: Performance of excessive health-related behaviors or maladaptive avoidance of medical care is a diagnostic criterion for Illness Anxiety Disorder.

Answer C: Evidence that symptoms are not being intentionally produced or feigned is not required for a DSM-5 diagnosis of Somatic Symptom Disorder.

The correct answer is: excessive thoughts, feelings, or behaviors related to the symptoms.

How well did you know this?
1
Not at all
2
3
4
5
Perfectly
25
Q

In the DSM-5, sleep terror is included:
Select one:

A.
with the Sleep-Wake Disorders as a separate diagnosis.

B.
as a type of Rapid Eye Movement Sleep Behavior Disorder.

C.
as a type of Non-Rapid Eye Movement Sleep Arousal Disorder.

D.
as a specifier for Nightmare Disorder.

A

Answer C is correct: In the DSM-5, sleep terror is a type of Non-Rapid Eye Movement Sleep Arousal Disorder. (The other type is sleepwalking.)

The correct answer is: as a type of Non-Rapid Eye Movement Sleep Arousal Disorder.

How well did you know this?
1
Not at all
2
3
4
5
Perfectly
26
Q

Drugs that interfere with the breakdown of __________ are used to treat memory loss and other cognitive problems in individuals with mild to moderate Alzheimer’s disease.
Select one:

A.
norepinephrine

B.
cortisol

C.
acetylcholine

D.
dopamine

A

Answer C is correct: Cholinesterase inhibitors (e.g., donepezil, galantamine, tacrine) are used to temporarily improve or slow the progression of memory loss and other cognitive impairments during the early stages of Alzheimer’s disease. These drugs prevent the breakdown of acetylcholine, which is important for learning, memory, and other cognitive functions.

Answers A, B, and D: See explanation for answer C.

The correct answer is: acetylcholine

How well did you know this?
1
Not at all
2
3
4
5
Perfectly
27
Q

The differential diagnosis of Major Depressive Disorder (pseudodementia) and mild Neurocognitive Disorder in older adults can be difficult because of the overlap in cognitive symptoms. However, the presence of which of the following suggests that Major Depressive Disorder is the appropriate diagnosis?
Select one:

A.
The onset of the patient’s cognitive symptoms was insidious.

B.
The severity of the patient’s cognitive symptoms increases in the evening.

C.
The patient seems unaware of his/her cognitive deficits.

D.
The patient is uncooperative during cognitive testing.

A

Answer D is correct: A lack of cooperation during testing is more characteristic of patients with depression than of those with a Neurocognitive Disorder who tend to be cooperative but inaccurate in their responses.

Answer A: In Major Depressive Disorder, symptom onset is usually sudden; but, in mild Neurocognitive Disorder, it is typically insidious (gradual and subtle).

Answer B: Increased severity of symptoms in the evening is characteristic of some forms of Neurocognitive Disorder but is uncommon in Major Depressive Disorder.

Answer C: A patient with Major Depressive Disorder is likely to complain about and exaggerate his/her cognitive problems, while a patient with a Neurocognitive Disorder often denies problems in the early stage of the disorder and is unaware of them in the later stages.

The correct answer is: The patient is uncooperative during cognitive testing.

How well did you know this?
1
Not at all
2
3
4
5
Perfectly
28
Q

Rosita R., age 32, maintains systematized paranoid delusions despite a lack of evidence for her beliefs. However, she shows almost no impairment in daily functioning other than some problems that are directly related to her delusions. The symptoms began six months ago shortly after she was fired from her job. The most likely DSM-5 diagnosis for Rosita is:
Select one:

A.
Paranoid Schizophrenia.

B.
Acute Stress Disorder.

C.
Conversion Disorder.

D.
Delusional Disorder.

A

Answer D is correct. Rosita’s delusions are consistent with Delusional Disorder because they are continuing despite a lack of evidence and because the effects of the delusions on her functioning are circumscribed (affect only those aspects of functioning that are directly related to the delusion).

Answer A: Paranoid Schizophrenia is not a DSM-5 diagnosis.

Answer B: A diagnosis of Acute Stress Disorder requires exposure to actual or threatened death, severe injury, or sexual violation in at least one of four ways and the presence of at least nine symptoms from any of five categories (i.e., intrusion, negative mood, dissociative symptoms, avoidance symptoms, arousal symptoms).

Answer C: A diagnosis of Conversion Disorder requires a disturbance in voluntary motor or sensory functioning that suggests a serious neurological or other medical condition (e.g., paralysis, seizures, blindness, loss of pain sensation) with evidence of an incompatibility between the symptom and recognized neurological and medical conditions.

The correct answer is: Delusional Disorder.

How well did you know this?
1
Not at all
2
3
4
5
Perfectly
29
Q

A DSM-5 diagnosis of Erectile Disorder requires the presence of characteristic symptoms for a minimum duration of approximately _____ months.
Select one:

A.
two

B.
six

C.
ten

D.

A

Answer B is correct: The DSM-5 requires the presence of characteristic symptoms for a minimum duration of about six months for all of the Sexual Dysfunctions except Substance/Medication-Induced Sexual Dysfunction for which no minimum duration is specified.

Answers A, C, and D: See explanation for answer B.

The correct answer is: six

How well did you know this?
1
Not at all
2
3
4
5
Perfectly
30
Q

The symptoms of Oppositional Defiant Disorder are categorized in three groups in the DSM-5. These groups are:
Select one:

A.
destruction of property, deceitfulness or theft, and serious violations of rules.

B.
negativistic, defiant, and hostile behavior.

C.
deceitfulness/dishonesty, irritability/aggressiveness, and failure to conform to social norms.

D.
angry/irritable mood, argumentative/defiant behavior, and vindictiveness.

A

Answer D is correct: Angry/irritable mood, argumentative/defiant behavior, and vindictiveness are the three categories of symptoms included in the DSM-5 for Oppositional Defiant Disorder.

Answer A: These are three of the four categories of symptoms included in the DSM-5 for Conduct Disorder (the fourth is aggression to people and animals).

Answers B and C: See explanation for answer D.

The correct answer is: angry/irritable mood, argumentative/defiant behavior, and vindictiveness.

How well did you know this?
1
Not at all
2
3
4
5
Perfectly
31
Q

According to Berry’s (1993) bidirectional acculturation model, ________ is characterized by low involvement in one’s own minority culture and in the majority culture.
Select one:

A.
moratorium

B.
alienation

C.
separation

D.
marginalization

A

Berry (1993) distinguishes between two independent dimensions in his discussion of acculturation – involvement in one’s minority culture and involvement in the majority culture.

a. Incorrect See explanation for response d.
b. Incorrect See explanation for response d.
c. Incorrect See explanation for response d.
d. CORRECT When a member of a minority group is uninvolved in both cultures, this is referred to as marginalization. (The other acculturation modes defined by Berry are integration, assimilation, and separation.)

The correct answer is: marginalization

How well did you know this?
1
Not at all
2
3
4
5
Perfectly
32
Q

Freud described countertransference as the analyst’s transference to the patient, which may interfere with the analyst’s functioning in therapy. More recently, psychoanalysts have begun to view countertransference as:
Select one:

A.
even more detrimental than Freud to the normal progress of therapy.

B.
an aid for the analyst for gaining insight into the patient’s inner world.

C.
an aid for the analyst in helping the client recognize how his/her behavior impacts on others.

D.
an opportunity for the analyst to respond to the client in an authentic way.

A

In recent decades, many psychoanalysts have broadened their view of countertransference. This shift can be traced to the work of Klein, Winnicott, and Kernberg who considered countertransference reactions to stem as much (or more) from the patient as from the analyst. (See G. O. Gabbard, Theories of personality and psychopathology: Psychoanalysis, in H. I. Kaplan and B. J. Sadock, Comprehensive textbook of psychiatry/VI, Baltimore,, Williams and Wilkins, 1995.)

a. Incorrect See explanation above.
b. CORRECT Countertransference is viewed by some psychoanalysts as the analyst’s total response to the client, a response that has as much to do with the client as with the analyst’s past. Consequently, the analyst’s countertransference feelings can give him/her some insight into the client’s processes.
c. Incorrect See explanation above.
d. Incorrect See explanation above.

The correct answer is: an aid for the analyst for gaining insight into the patient’s inner world.

How well did you know this?
1
Not at all
2
3
4
5
Perfectly
33
Q

Researchers interested in evaluating the outcomes of psychotherapy distinguish between effectiveness and efficacy research. In contrast to effectiveness research, efficacy research:
Select one:

A.
has better internal validity but limited external validity.

B.
has limited internal validity but better external validity.

C.
has better internal and external validity.

D.
has limited internal and external validity.

A

Efficacy studies are conducted in well-controlled conditions, often using a structured manualized session format. In contrast, effectiveness studies are conducted in real-world settings with less experimental control. (Note that, to identify the correct answer to this question, you not only have to be familiar with efficacy and effectiveness research but also with internal and external validity, which are described in the Statistics and Research Design chapter of the written study materials.)

a. CORRECT A criticism of efficacy research is that, by controlling many of the crucial elements of what is actually done during the course of delivering psychotherapy, the results have good internal validity but limited external validity. Therefore, while efficacy studies are useful for determining whether a treatment has beneficial effects under controlled conditions, effectiveness studies are useful for determining if those effects generalize to other conditions.
b. Incorrect See explanation for response a.
c. Incorrect See explanation for response a.
d. Incorrect See explanation for response a.

The correct answer is: has better internal validity but limited external validity.

How well did you know this?
1
Not at all
2
3
4
5
Perfectly
34
Q

The belief that a child’s misbehavior has one of four goals - i.e., attention, revenge, power, or to display inadequacy - is most consistent with:
Select one:

A.
Becks cognitive-behavioral therapy.

B.
Adlers individual psychology.

C.
Perlss Gestalt therapy.

D.
Mahlers object relations theory.

A

Adler believed that all behaviors are goal-directed and purposeful.

a. Incorrect See explanation for response b.
b. CORRECT Dreikurs, an associate of Adler’s, described the misbehavior of children as attempts to belong, which reflect faulty beliefs about what it takes to belong (e.g., “I belong only when I”m the center of attention”).
c. Incorrect See explanation for response b.
d. Incorrect See explanation for response b.

The correct answer is: Adlers individual psychology.

How well did you know this?
1
Not at all
2
3
4
5
Perfectly
35
Q

Asking the “miracle question” is an initial intervention in which type of therapy?
Select one:

A.
transtheoretical

B.
interpersonal

C.
REBT

D.
solution-focused

A

The “miracle question” is used to help clients identify solutions to their problems.

a. Incorrect See explanation for response d.
b. Incorrect See explanation for response d.
c. Incorrect See explanation for response d.
d. CORRECT As its name implies, solution-focused therapy focuses on identifying solutions to problems. The miracle question is one of the techniques used by solution-focused therapists to identify solutions and establish therapy goals. It involves asking the client how the client’s life would be if his/her problems were suddenly resolved.

The correct answer is: solution-focused

How well did you know this?
1
Not at all
2
3
4
5
Perfectly
36
Q

For a Gestalt therapist, a primary goal of treatment is to help the client:
Select one:

A.
integrate the present with his/her past and future.

B.
integrate the various aspects of the self.

C.
develop a success identity.

D.
develop a healthy style of life.

A

For the exam, you want to know that the primary goals of Gestalt therapy are to increase awareness and integration of all aspects of the self.

a Incorrect In Gestalt therapy, the focus is on the present. In fact, its founder, Fritz Perls, argued that “nothing exists but the now.”

b. CORRECT A primary goal of gestalt therapy is to integrate all aspects of the self; i.e., one’s feelings, thoughts, and actions.

c Incorrect This is the goal of reality therapy.

d Incorrect This sounds more like Adlerian therapy.

The correct answer is: integrate the various aspects of the self.

How well did you know this?
1
Not at all
2
3
4
5
Perfectly
37
Q

Research on Helms’s White Racial Identity Development Model suggests that a White therapist will usually be most successful when working with a client from an ethnic/racial minority group when the therapist is in which stage?
Select one:

A.
reintegration

B.
immersion-emersion

C.
autonomy

D.
integrative awareness

A

Helms’s White Racial Identity Development Model distinguishes between six identity statuses (stages): contact, disintegration, reintegration, pseudo-independence, immersion-emersion, and autonomy. For the exam, you’ll want to be familiar with the characteristics of each stage.

a. Incorrect See explanation for response c.
b. Incorrect See explanation for response c.
c. CORRECT Perhaps not surprisingly, a White therapist is likely to be most effective when working with a client from an ethnic/racial minority group when the therapist is in the final stage of White identity development - i.e., the autonomy stage. A person in this stage has internalized a positive (nonracist) White identity that includes appreciation of and respect for racial/cultural similarities and differences.
d. Incorrect See explanation for response c.

The correct answer is: autonomy

How well did you know this?
1
Not at all
2
3
4
5
Perfectly
38
Q

The primary goal of a newly developed community-based mental health program is to help people recently released from a psychiatric hospital adjust to life in the community. This is an example of:
Select one:

A.
primary prevention.

B.
secondary prevention.

C.
tertiary prevention.

D.
crisis intervention.

A

Prevention programs are often described as primary, secondary, or tertiary, depending on their goals. You’re likely to encounter a question or two on prevention on the exam, so you’ll want to be familiar with the characteristics of the three types – and these are described in the Clinical Psychology chapter of the written study materials.

a. Incorrect Primary preventions are designed to prevent the development of a mental disorder.
b. Incorrect Secondary preventions are designed to reduce the prevalence of mental disorders through early identification and intervention.
c. CORRECT Tertiary preventions are designed to reduce the prevalence of mental disorders by reducing their duration or preventing their recurrence.
d. Incorrect Crisis interventions are designed to reduce the negative consequences of crises.

The correct answer is: tertiary prevention.

How well did you know this?
1
Not at all
2
3
4
5
Perfectly
39
Q

In contrast to traditional approaches to psychotherapy, culturally sensitive approaches attempt to understand a client’s experience of an illness within the client’s cultural context. In other words, culturally sensitive approaches adopt an ______ perspective.
Select one:

A.
emic

B.
etic

C.
emetic

D.
endogenous

A

The terms “emic” and “etic” were originally used by linguistic anthropologists but are now also used by psychologists interested in cross-cultural psychotherapy.

a. CORRECT Traditional psychotherapies reflect an etic approach (“view from the outside”), while culturally sensitive therapies emphasize an emic (“view from the inside”) approach.
b. Incorrect See explanation for response b.
c. Incorrect See explanation for response b.
d. Incorrect See explanation for response b.

The correct answer is: emic

How well did you know this?
1
Not at all
2
3
4
5
Perfectly
40
Q

Margaret Mahler proposed that the development of a sense of self is related to:
Select one:

A.
separation-individuation.

B.
projective identification.

C.
pseudomutuality.

D.
assimilation-accommodation.

A

Mahler’s theory of early development focuses on the processes that contribute to the development of a sense of identity.

a. CORRECT Separation-individuation is triggered by the child’s ability to separate from his/her primary caregiver. It begins at about 4 to 5 months of age when a child who is being held by his/her caregiver is able to lean away to scan the environment.
b. Incorrect Melanie Klein used the term projective identification to refer to projecting disliked or undesirable aspects of the internal object (self) onto an external object (caregiver).
c. Incorrect Pseudomutuality describes a relationship between family members that appears to be healthy (is open and mutually empathic) but is not.
d. Incorrect Assimilation and accommodation are terms used by Piaget to describe the modification and development of cognitive schemas.

The correct answer is: separation-individuation.

How well did you know this?
1
Not at all
2
3
4
5
Perfectly
41
Q

Carl Jung believed that a client’s transference:
Select one:

A.
is a fantasy that distracts the client from reality.

B.
represents mixed feelings toward the therapist.

C.
is a form of “acting out.”

D.
reflects the clients personal and collective unconscious.

A

The correct answer to this question should have been easy to identify if you’re at all familiar with Jung”s work, since only one of the answers includes language that is associated with his analytical psychology.

a. Incorrect See explanation for response d.
b. Incorrect See explanation for response d.
c. Incorrect See explanation for response d.
d. CORRECT Jung viewed transference as the projection of both the personal and collective unconscious.

The correct answer is: reflects the clients personal and collective unconscious.

How well did you know this?
1
Not at all
2
3
4
5
Perfectly
42
Q

Smith, Glass, and Miller’s (1980) meta-analysis of the psychotherapy outcome research found that people receiving therapy are “better off” than about _____% of people who need treatment but do not receive it.
Select one:

A.
45

B.
55

C.
80

D.
95

A

For the exam, you want to be familiar with the results of the Smith et al. meta-analysis as well as understand what meta-analysis is used for and be able to interpret an effect size. These are described in the Clinical Psychology chapter of the written study materials.

a. Incorrect See explanation for response c.
b. Incorrect See explanation for response c.
c. CORRECT Smith, Glass, and Miller (1980) report an average effect size of .85 for 475 controlled studies of psychotherapy. This means that the mean outcome score of people who receive treatment is higher than the mean outcome score of about 80% of people who need treatment but do not receive it.
d. Incorrect See explanation for response c.

The correct answer is: 80

How well did you know this?
1
Not at all
2
3
4
5
Perfectly
43
Q

According to Howard et al.’s (1996) phase model, which of the following is most likely to be affected during the first few sessions of psychotherapy?
Select one:

A.
coping skills

B.
severity of symptoms

C.
insight

D.
feelings of hopelessness

A

Howard et al.’s (1996) phase model predicts that the effects of therapy vary, depending on the number of sessions. The characteristics of the three phases of the model (remoralization, remediation, and rehabilitation) are described in the Clinical Psychology chapter of the written study materials.

a. Incorrect See explanation for response d.
b. Incorrect See explanation for response d.
c. Incorrect See explanation for response d.
d. CORRECT Remoralization occurs during the first few therapy sessions and is characterized by a decline in feelings of hopelessness (K. Howard et al., Evaluation of psychotherapy: Efficacy, effectiveness, and patient progress, American Psychologist, 51, 1059-1064, 1996).

The correct answer is: feelings of hopelessness

How well did you know this?
1
Not at all
2
3
4
5
Perfectly
44
Q

In client-centered case consultation, the consultant’s primary goal is to:
Select one:

A.
help the consultee determine how to work more effectively with a particular client.

B.
help the consultee identify methods for evaluating his/her own professional services.

C.
help a consultee enhance his/her skills and knowledge so that the consultee functions more effectively in the future.

D.
work collaboratively with the consultee to jointly deliver an appropriate intervention to a client.

A

Gerald Caplan (1970) distinguishes between four types of mental health consultation: client-centered case consultation, consultee-centered case consultation, program-centered administrative consultation, and consultee-centered administrative consultation.

a. CORRECT Client-centered case consultation focuses on a particular client. In this type of consultation, the consultant collects information about the client and makes recommendations to the consultee about the best course of action.
b. Incorrect This does not accurately describe the primary goal of client-centered case consultation.
c. Incorrect This describes consultee-centered case consultation.
d. Incorrect This describes “collaboration” rather than “consultation. “

The correct answer is: help the consultee determine how to work more effectively with a particular client.

How well did you know this?
1
Not at all
2
3
4
5
Perfectly
45
Q

In comparing Asian and Asian-American therapy clients to Anglo clients, the experts frequently point out that the former:
Select one:

A.
tend to prefer a less directive therapeutic approach.

B.
are grounded more in the here-and-now than in the past or the future.

C.
respond better when goal-setting is delayed.

D.
are likely to express emotional problems as somatic symptoms.

A

Although it is important not to overgeneralize about members of any particular group, the literature on cross-cultural psychotherapy does provide some general conclusions about clients belonging to different ethnic/cultural groups.

a. Incorrect This is the opposite of what is true.
b. Incorrect In terms of time perspective, individuals from Asian cultures usually place importance on the relationship of the past to the present.
c. Incorrect It is usually best to identify goals early in therapy (even during the first session) when working with Asian clients.
d. CORRECT People from Asian cultures ordinarily view the mind and body as inseparable and, as a result, often express mental health problems as somatic complaints such as headaches, fatigue, or sleep problems.

The correct answer is: are likely to express emotional problems as somatic symptoms.

How well did you know this?
1
Not at all
2
3
4
5
Perfectly
46
Q

From the perspective of feminist therapy, therapist self-disclosure is:
Select one:

A.
contraindicated because it puts the client in a passive role.

B.
necessary during the early stages of therapy to encourage the client’s participation.

C.
a means of fostering a special bond between the client and the therapist.

D.
useful for promoting an egalitarian relationship between the therapist and client.

A

A key characteristic of feminist therapy is its emphasis on an egalitarian relationship.

a. Incorrect Promoting a passive role in a client is discouraged in feminist therapy, but therapist self-disclosure (if done appropriately) is not considered a contributor to passivity.
b. Incorrect This describes one use of self-disclosure, but this response doesn”t best describe the role of self-disclosure in feminist therapy.
c. Incorrect Feminist therapists generally discourage developing a “special bond” because doing so can foster the client’s dependence on the therapist.
d. CORRECT Feminist therapists view sharing their own life experiences with clients as a way of promoting an egalitarian relationship.

The correct answer is: useful for promoting an egalitarian relationship between the therapist and client.

How well did you know this?
1
Not at all
2
3
4
5
Perfectly
47
Q

The first stage in Cross’s (2001) Black Racial Identity Development Model is:
Select one:

A.
conformity.

B.
incorporation.

C.
contact.

D.
pre-encounter.

A

The most recent version of Cross’s Black Racial Identity Development Model distinguishes between four stages: pre-encounter, encounter, immersion-emersion, and internalization.

a. Incorrect See explanation for response d.
b. Incorrect See explanation for response d.
c. Incorrect See explanation for response d.
d. CORRECT Cross’s model is based on the premise that African American identity development is directly linked to racial oppression. During the initial pre-encounter stage, race has low salience.

The correct answer is: pre-encounter.

How well did you know this?
1
Not at all
2
3
4
5
Perfectly
48
Q

Replacing a “failure identity” with a “success identity” is a goal of treatment for practitioners of _______ therapy.
Select one:

A.
Gestalt

B.
reality

C.
solution-focused

D.
Adlerian

A

Of the therapies listed, only one distinguishes between success and failure identities.

a. Incorrect See explanation for response b.
b. CORRECT Glasser, the founder of reality therapy, viewed identity as a basic psychological need and distinguished between success and failure identities. A person develops a success identity when the person fulfills his/her other needs in a responsible way.
c. Incorrect See explanation for response b.
d. Incorrect See explanation for response b.

The correct answer is: reality

How well did you know this?
1
Not at all
2
3
4
5
Perfectly
49
Q

Prochaska and DiClemente’s (1982) stages of change (transtheoretical) model predicts that a person in the __________ stage plans to take action within the next six months that will alter his/her problematic behavior.
Select one:

A.
action

B.
contemplation

C.
preparation

D.
precontemplation

A

For the exam, you want to be familiar with the six stages of Prochaska and DiClemente’s stages of change model: precontemplation, contemplation, preparation, action, maintenance, and termination. The stages are described in the Clinical Psychology chapter of the written study materials.

a. Incorrect A person in this stage is currently taking actions to alter his/her behavior.
b. CORRECT A person in the contemplation stage intends to take action in the next six months.
c. Incorrect A person in the preparation stage is planning to take action in the near future (usually defined as in the next month).
d. Incorrect A person in this stage is not planning to change in the foreseeable future.

The correct answer is: contemplation

How well did you know this?
1
Not at all
2
3
4
5
Perfectly
50
Q

Atkinson, Morten, and Sue’s (1993) Racial/Cultural Identity Development Model proposes that people in which of the following stages begin to question their rejection of all aspects of the dominant culture and absolute loyalty to their own culture?
Select one:

A.
encounter

B.
dissonance

C.
integrative awareness

D.
introspection

A

The Racial/Cultural Identity Development Model distinguishes between five stages: conformity, dissonance, resistance and immersion, introspection, and integrative awareness.

a. Incorrect The encounter stage is the second stage in Cross’s (1991) Black Racial (Nigrescence) Identity Development Model.
b. Incorrect During the dissonance stage, the individual begins to question conformity to the dominant culture.
c. Incorrect During the integrative awareness stage, the person bases acceptance or rejection of aspects of the majority and minority cultures on an objective evaluation.
d. CORRECT During the introspection stage, the individual begins to question the unequivocal position that he/she adopted during the previous resistance-immersion stage with regard to his/her own culture and the dominant culture.

The correct answer is: introspection

How well did you know this?
1
Not at all
2
3
4
5
Perfectly
51
Q

A family therapist using the structural approach of Salvador Minuchin would most likely:
Select one:

A.
clarify boundaries between family members in order to reduce enmeshment.

B.
work initially with the most differentiated family member.

C.
use a multiple-therapist team to prevent any one therapist from becoming “triangulated” into the family system.

D.
issue specific “directives” designed to counteract dysfunctional processes.

A

As its name implies, structural family therapy focuses on altering the family”s structure in order to change the behavior patterns of family members.

a. CORRECT Even if you are unfamiliar with Minuchin, you may have been able to guess that structural therapy would be concerned with boundaries. Structural family therapists view family dysfunction as being related to boundaries that are too diffuse (enmeshed) or too rigid (disengaged).
b. Incorrect This is more characteristic of Bowen’s approach to family therapy.
c. Incorrect Minuchin”s approach does not involve the use of multiple-therapist teams.
d. Incorrect The issuance of directives is more characteristic of strategic family therapy than of structural family therapy.

The correct answer is: clarify boundaries between family members in order to reduce enmeshment.

How well did you know this?
1
Not at all
2
3
4
5
Perfectly
52
Q

According to Irvin Yalom, __________ in group therapy is the analogue of the therapist-client relationship in individual therapy.
Select one:

A.
universality

B.
identification

C.
cohesiveness

D.
alliance

A

Yalom argues that the therapeutic relationship is the sine qua non of effective individual therapy and, therefore, that there must be a comparable phenomenon in group therapy.

a. Incorrect See explanation for response c.
b. Incorrect See explanation for response c.
c. CORRECT Yalom uses the term cohesiveness to refer to the various relationships that occur in a group - i.e., the relationships between group members and between each member and the therapist and the relationship between each member and the group itself. For Yalom, cohesiveness is the primary mode of help in group therapy, with highly cohesive groups having better outcomes.
d. Incorrect See explanation for response c.

The correct answer is: cohesiveness

How well did you know this?
1
Not at all
2
3
4
5
Perfectly
53
Q

According to Freud’s notion of __________, dreams and slips of the tongue are meaningful goal-directed phenomena.
Select one:

A.
psychic dynamogenesis

B.
reciprocal determinism

C.
psychic determinism

D.
ptyalism

A

Freud interpreted dreams, slips of the tongue, etc. as manifestations of unconscious material.

a. Incorrect Dynamogenesis is the principle that motor responses are proportional to sensory activities.
b. Incorrect Reciprocal determinism proposes that behavior, person, and situation interact and influence each other.
c. CORRECT According to Freud”s doctrine of psychic determinism, behaviors are not the result of free will but, instead, are caused by unconscious phenomena.
d. Incorrect Ptyalism refers to the excessive production of saliva, something Pavlov, not Freud, would be interested in.

The correct answer is: psychic determinism

How well did you know this?
1
Not at all
2
3
4
5
Perfectly
54
Q

The information that family members continuously exchange and that helps minimize deviation and maintain the family’s stability is referred to as ________ feedback.
Select one:

A.
external

B.
internal

C.
negative

D.
positive

A

In family systems theory, the information exchange between family members can act as either positive or negative feedback.

a. Incorrect See explanation for response c.
b. Incorrect See explanation for response c.
c. CORRECT Negative feedback is corrective and helps the system return to or maintain its steady state. Thus, a “negative feedback loop” helps minimize deviation and thereby maintains the family’s stability.
d. Incorrect Positive feedback increases deviation from a steady state and, therefore, produces a change in the family’s functioning.

The correct answer is: negative

How well did you know this?
1
Not at all
2
3
4
5
Perfectly
55
Q

When working with an African American family, it is important to keep in mind that:
Select one:

A.
African American families are typically matriarchal.

B.
African American husbands are usually less tolerant of the career efforts of their wives than White husbands are.

C.
African American males and females do not adhere as rigidly to gender stereotypes as White males and females do.

D.
African American wives are more willing than White wives to let their husbands make domestic decisions.

A

Research on African American families has generally found that African American men and women are less concerned about sex-role stereotypes than White men and women. However, the power structure of the African American family is not substantially different from that found in many White families.

a. Incorrect A number of investigators have pointed out that African American and White husbands exercise the same degree of family decision-making power and that the matriarchal African American family is, in fact, a myth.
b. Incorrect If anything, the opposite is true because of their de-emphasis on sex-role stereotypes and social conditions that have made it easier for African American women to succeed educationally and economically than African American men.
c. CORRECT According to Billingsley, an important aspect of the cultural heritage of African American families is a de-emphasis on rigid sex-linked stereotypes (Black Families in White America, Englewood Cliffs: Prentice-Hall, 1969).
d. Incorrect In terms of decision-making, there does not seem to be much difference between African American and White families.

The correct answer is: African American males and females do not adhere as rigidly to gender stereotypes as White males and females do.

How well did you know this?
1
Not at all
2
3
4
5
Perfectly
56
Q

In the context of family therapy, the purpose of “reframing” is to:
Select one:

A.
diffuse blame.

B.
redefine behaviors.

C.
increase specificity.

D.
anchor behaviors.

A

Even if you’ve never heard of “reframing,” you probably could have guessed that its purpose is to redefine the situation.

a. Incorrect See explanation for response b.
b. CORRECT As its name implies, reframing is done in order to help a client see things from a different perspective. It is often used in a paradoxical way – e.g., telling an adolescent that “your parents’ nagging is a sign of their love for you.”
c. Incorrect See explanation for response b.
d. Incorrect See explanation for response b.

The correct answer is: redefine behaviors.

How well did you know this?
1
Not at all
2
3
4
5
Perfectly
57
Q

According to the Publication Manual of the American Psychological Association, which of the following statements is an acceptable description of a research participant?
Select one:

A.
non-white participant

B.
disabled participant

C.
participant diagnosed with Borderline Personality Disorder

D.
participant suffering from multiple sclerosis

A

Ways to reduce bias in language are addressed in the APA’s Publication Manual.

a. Incorrect Non-white is improper. The actual race/ethnicity is preferable.
b. Incorrect “Participant with a disability” would be more appropriate.
c. CORRECT This is an acceptable way to describe an individual who has a mental disorder.
d. Incorrect The word “suffering” is inappropriate. “A person with (or who has) multiple sclerosis” would be better.

The correct answer is: participant diagnosed with Borderline Personality Disorder

How well did you know this?
1
Not at all
2
3
4
5
Perfectly
58
Q

Vicarious liability is most likely to be an issue when a psychologist is acting in which of the following capacities?
Select one:

A.
faculty member

B.
supervisor

C.
mentor

D.
therapist

A

Vicarious liability (also known as derivative and secondary liability) is a legal term that refers to a person’s responsibility for the actions of another person.

a. Incorrect See explanation for response b.
b. CORRECT A psychologist may have vicarious liability when his/her employee or supervisee engages in illegal behavior.
c. Incorrect See explanation for response b.
d. Incorrect See explanation for response b.

The correct answer is: supervisor

How well did you know this?
1
Not at all
2
3
4
5
Perfectly
59
Q

When serving as an evaluator in a child custody case, a psychologist’s primary concern should be:
Select one:

A.
what the child desires.

B.
maintaining a position of neutrality.

C.
the best interests of the child.

D.
the best interests of the person who hired the psychologist.

A

This issue is addressed in the Guidelines for Child Custody Evaluations in Family Law Proceedings, which is summarized in the chapter on ethics and professional issues in the written study materials.

a. Incorrect See explanation for response c.
b. Incorrect See explanation for response c.
c. CORRECT Regardless of who employed the psychologist, the child’s best interests (which may or may not be consistent with the child’s desires) are a priority, and a psychologist should make this clear to all parties at the outset.
d. Incorrect See explanation for response c.

The correct answer is: the best interests of the child.

How well did you know this?
1
Not at all
2
3
4
5
Perfectly
60
Q

Whenever her clients cancel an appointment with Dr. Penny Pincher within less than 24 hours of the appointment, Dr. Pincher routinely bills the client’s insurance company for her full hourly fee without informing the company that the client missed the appointment. This practice is:
Select one:

A.
illegal and unethical.

B.
illegal but ethical.

C.
legal and ethical.

D.
legal but unethical.

A

For questions about insurance coverage, always choose the answer that is most consistent with legal and ethical requirements.

a. CORRECT The practice described in this question represents insurance fraud, which is both illegal and unethical. Billing an insurance company for missed appointments would be acceptable only if the company agreed to this practice (which is, of course, unlikely). See, e.g., Standard 6.06 of the APA’s Ethics Code.
b. Incorrect See explanation for response a.
c. Incorrect See explanation for response a.
d. Incorrect See explanation for response a.

The correct answer is: illegal and unethical.

How well did you know this?
1
Not at all
2
3
4
5
Perfectly
61
Q

A former client owes you over $600.00 in therapy fees and you are considering using a collection agency to collect the money she owes you. As an ethical psychologist, you should:
Select one:

A.
decide not to do so because using a collection agency is prohibited by the ethical guidelines.

B.
do so only if you had informed the client at the beginning of therapy that you would do so if she did not pay her fees in a timely manner.

C.
contact the client first to inform her of your intent to use a collection agency if she does not pay her outstanding fees by a specific date.

D.
use a collection agency only as a last resort.

A

The use of a collection agency is explicitly addressed by Standard 6.04 of the APA’s Ethics Code and indirectly addressed by Principles I.12 and III.14 of the Canadian Code of Ethics and is discussed in the chapter on ethics and professional issues in the written study materials.

a. Incorrect Use of a collection agency is not prohibited by ethical guidelines.
b. Incorrect Informing the client of your policy at the beginning of therapy is a good idea but is not sufficient.
c. CORRECT Psychologists always want to treat clients fairly, and the best policy would be to send a letter to the client informing her of your intent to use a collection agency prior to actually doing so. Informing the client is consistent with the provisions of Standard 6.04(e) of the Ethics Code, which states that if psychologists plan to use a collection agency or other legal means to collect unpaid fees, they “first inform the person that such measures will be taken and provide that person an opportunity to make prompt payment.”
d. Incorrect This answer does not address your ethical obligation in this situation.

The correct answer is: contact the client first to inform her of your intent to use a collection agency if she does not pay her outstanding fees by a specific date.

How well did you know this?
1
Not at all
2
3
4
5
Perfectly
62
Q

You have been seeing Leticia Lopez in therapy for several months. Leticia is 24 years old and lives with her widowed mother, who is paying for Leticia’s therapy. One day, Leticia”s mother calls and says she is very concerned about Leticia and wants to know what she can do to help Leticia feel better about herself. Mrs. Lopez asks that you not tell Leticia that she has called. Your best course of action in this situation would be to:
Select one:

A.
give Mrs. Lopez the specific advice she has requested.

B.
tell Mrs. Lopez to ask Leticia what she (Mrs. Lopez) can do to help her.

C.
tell Mrs. Lopez that it would be best if you discussed this matter with Leticia.

D.
suggest that Mrs. Lopez accompany Leticia to her next therapy session.

A

Leticia is an adult and, even though her mother is paying for her therapy, this does not limit Leticia’s confidentiality.

a. Incorrect Discussing Leticia’s status with her mother without Leticia’s consent would represent a breach of confidentiality.
b. Incorrect This alternative does not explicitly violate ethical guidelines, but response c is a better answer.
c. CORRECT This course of action is most consistent with ethical guidelines regarding confidentiality and is also likely to be in the best interests of Leticia from a clinical perspective.
d. Incorrect This would not be the most appropriate course of action from either an ethical or clinical perspective.

The correct answer is: tell Mrs. Lopez that it would be best if you discussed this matter with Leticia.

How well did you know this?
1
Not at all
2
3
4
5
Perfectly
63
Q

Which of the following best describes ethical guidelines regarding sexual intimacies with former therapy clients?
Select one:

A.
Psychologists are prohibited from having sexual intimacies with former therapy clients under any circumstances.

B.
Psychologists are prohibited from having sexual intimacies with former therapy clients for at least one year following the termination of therapy.

C.
Psychologists are prohibited from having sexual intimacies with former therapy clients in certain circumstances (e.g., when there is a risk of exploitation).

D.
Psychologists are not prohibited from having sexual intimacies with former therapy clients.

A

The APA’s Ethics Code and the Canadian Code of Ethics both discourage psychologists from becoming sexually involved with former therapy clients.

a. Incorrect See explanation for response c.
b. Incorrect See explanation for response c.
c. CORRECT Standard 10.08 of the APA’s Ethics Code states that sexual intimacies with former therapy clients may be acceptable two years after termination of therapy when certain conditions are met (e.g., when there is no exploitation of the client), while Principle II.27 of the Canadian Code of Ethics states that sexual intimacies with former therapy clients must be avoided when “the power relationship reasonably could be expected to influence the client’s decision making.”
d. Incorrect See explanation for response c.

The correct answer is: Psychologists are prohibited from having sexual intimacies with former therapy clients in certain circumstances (e.g., when there is a risk of exploitation).

How well did you know this?
1
Not at all
2
3
4
5
Perfectly
64
Q

In most situations, the “holder of the privilege” is:
Select one:

A.
the therapist.

B.
the client.

C.
the therapist and client jointly.

D.
the court.

A

Privilege is a legal term that refers to a client’s confidentiality in the context of a legal proceeding. Privilege is addressed in the discussion of privacy and confidentiality in the Ethics Manual of the written study materials.

a. Incorrect See explanation for response b.
b. CORRECT Privilege is a legal requirement that prohibits (with some exceptions) confidential client information from being disclosed in legal proceedings. The client is ordinarily the holder of the privilege - i.e., in most circumstances, only the client can determine when confidential information may be disclosed.
c. Incorrect See explanation for response b.
d. Incorrect See explanation for response b.

The correct answer is: the client.

How well did you know this?
1
Not at all
2
3
4
5
Perfectly
65
Q

Dr. Jones, a clinical psychologist, has been seeing Lisa L. in therapy for one month. Dr. Jones considers Lisa to be a very attractive woman and finds himself having sexual fantasies about her. As an ethical psychologist, Dr. Jones should:
Select one:

A.
recognize that such attraction is normal but proceed with caution.

B.
confess his attraction to Lisa and make it a topic for mutual examination in therapy.

C.
recognize that such attraction is inappropriate and refer Lisa to another therapist without explaining the reason to her.

D.
continue working with Lisa but consult with a colleague if he believes his attraction might be interfering with his objectivity.

A

As with many exam questions that pose an ethical dilemma, the best answer to this one is the answer that describes the most conservative (cautious) action.

a. Incorrect Attraction to a client is not necessarily “normal” and “proceeding with caution” may not be adequate. Therefore, response d is a better answer.
b. Incorrect Whether an attraction toward a client should ever be discussed with the client is a controversial issue among therapists. In most cases, however, it is not advisable.
c. Incorrect Referring Lisa to another therapist might be the appropriate action if Dr. Jones feels that his attraction to Lisa is interfering with his ability to treat her. The information given in the question, however, does not suggest that this is the case, and response d is a better answer.
d. CORRECT The most professional and ethical behavior in this situation would be for Dr. Jones to recognize his feelings while realizing that, as a professional, he cannot act on those feelings. It would be acceptable, under these conditions, for Dr. Jones to continue working with Lisa unless his feelings begin to interfere with his ability to provide treatment to her. If this occurs, Dr. Jones should seek professional consultation to determine the best course of action, and doing so would be consistent with the provisions of Standard 2.06 of the APA’s Ethics Code.

The correct answer is: continue working with Lisa but consult with a colleague if he believes his attraction might be interfering with his objectivity.

How well did you know this?
1
Not at all
2
3
4
5
Perfectly
66
Q

The primary function of the psychology licensing board is best described as:
Select one:

A.
limiting access to the profession.

B.
providing sanctions for unethical and illegal behavior on the part of psychologists.

C.
protecting the public welfare.

D.
accrediting graduate programs in psychology.

A

The primary goal of the psychology licensing board is the protection of the public.

a. Incorrect See explanation for response c.
b. Incorrect See explanation for response c.
c. CORRECT Psychology licensing boards protect the public by creating and enforcing regulations designed to ensure that psychologists meet minimal levels of competence. While the boards sanction psychologists for illegal and unethical behavior (answer b), this is only one of a number of functions that serve the overall goal of protecting public welfare.
d. Incorrect See explanation for response c.

The correct answer is: protecting the public welfare.

How well did you know this?
1
Not at all
2
3
4
5
Perfectly
67
Q

Which of the following best describes ethical guidelines for the use of deception in research?
Select one:

A.
Deception is prohibited in all circumstances.

B.
Deception is prohibited except when the purpose and design of the study require the use of unobtrusive measures.

C.
Deception is prohibited whenever it involves deceiving potential participants about aspects of the study that would affect their willingness to participate.

D.
Deception is acceptable only when participants have given limited consent after being told about the general nature of the study.

A

The use of deception in research is addressed in Standard 8.07 of the APA’s Ethics Code and Principles III.23, 24, and 27 of the Canadian Code of Ethics and is discussed in the chapter on ethics and professional issues in the written study materials.

a. Incorrect See explanation for response c.
b. Incorrect See explanation for response c.
c. CORRECT According to ethical guidelines, deception can be used in research only when it is justified by the study”s potential value, when it has been determined that alternative procedures are not available, and when participants are not deceived about aspects of a study that would otherwise influence their willingness to participate in the study.
d. Incorrect See explanation for response c.

The correct answer is: Deception is prohibited whenever it involves deceiving potential participants about aspects of the study that would affect their willingness to participate.

How well did you know this?
1
Not at all
2
3
4
5
Perfectly
68
Q

Which of the following statements best describes a researcher’s obligations regarding the use of animals in psychological research?
Select one:

A.
Animals should never be used in research when alternative procedures are available.

B.
Animals should never be used in research if a research procedure will cause them pain or stress.

C.
Inflicting pain or stress on animals is acceptable in research only when an alternative (nonpainful) procedure is unavailable and the use of the procedure is justified by the potential value of the study.

D.
Inflicting pain or stress on animals is acceptable in research as long as it is minimized and the animals life is terminated as soon as is feasible.

A

Standard 8.09 of the APA’s Ethics Code and Principle II.46 of the Canadian Code of Ethics address the use of animals in research.

a. Incorrect See explanation for response c.
b. Incorrect See explanation for response c.
c. CORRECT Standard 8.09(e) of the Ethics Code states that “psychologists use a procedure subjecting animals to pain, stress, or privation only when an alternative procedure is unavailable and the goal is justified by its prospective scientific, educational, or applied value.” Principle II.46 of the Canadian Code of Ethics provides a nearly identical guideline.
d. Incorrect See explanation for response c.

The correct answer is: Inflicting pain or stress on animals is acceptable in research only when an alternative (nonpainful) procedure is unavailable and the use of the procedure is justified by the potential value of the study.

How well did you know this?
1
Not at all
2
3
4
5
Perfectly
69
Q

Dr. Sam Solo is the only psychologist in a small town, and his son is enrolled in the town’s only elementary school. Dr. Solo receives a call from his son’s teacher who says she would like to begin therapy with him. As an ethical psychologist, Dr. Solo should:
Select one:

A.
tell the teacher that he is ethically prohibited from seeing her because to do so would constitute a multiple (dual) relationship.

B.
begin therapy with her and discuss the potential for conflicts during the initial session and, if necessary, in subsequent sessions.

C.
begin therapy with her but seek consultation if any problems arise.

D.
begin therapy with her but take special precautions to ensure that client confidentiality is maintained.

A

Multiple relationships are addressed in Standard 3 of the APA’s Ethics Code and are discussed in the Ethics Manual. Keep in mind for the licensing exam that the prohibition against multiple relationships is not absolute.

a. Incorrect See explanation for response b.
b. CORRECT The ethical guidelines discourage multiple relationships whenever there is a potential for exploitation or loss of objectivity. The prohibition against multiple relationships is not absolute, however, so if Dr. Solo is the only psychologist in town, seeing the teacher and discussing the potential for conflict is the best choice. A better course of action might be to see if the teacher can be treated by a therapist in a neighboring town, but this option is not provided by any of the answers.
c. Incorrect See explanation for response b.
d. Incorrect Confidentiality is not the only issue of concern in multiple relationship situations, so this is not the best answer of those given.

The correct answer is: begin therapy with her and discuss the potential for conflicts during the initial session and, if necessary, in subsequent sessions.

How well did you know this?
1
Not at all
2
3
4
5
Perfectly
70
Q

Dr. Harold Hanson, a psychologist who heads the personnel committee at a mental health facility, recommends that a psychologist who has an unresolved charge of sexual harassment against him not be considered for a promotion. In terms of his ethical responsibilities, Dr. Hanson:
Select one:

A.
has acted ethically since sexual harassment is explicitly prohibited by the ethical guidelines.

B.
has acted ethically as long as he is willing to consider the psychologist for promotion if he is acquitted of the harassment charge.

C.
has acted ethically as long as his decision is consistent with the stated promotion policy of the mental health facility.

D.
has acted unethically by violating the requirements of the ethical guidelines regarding employment procedures.

A

This situation is explicitly addressed by Standard 1.08 of the APA’s Ethics Code and indirectly addressed by Principle I.13 of the Canadian Code of Ethics.

a. Incorrect See explanation for response d.
b. Incorrect See explanation for response d.
c. Incorrect See explanation for response d.
d. CORRECT Standard 1.08 of the APA’s Ethic Code prohibits denying a person a promotion “based solely upon their having made or their being the subject of an ethics complaint.” Note that this prohibition does not necessarily preclude doing so when a charge against a person has been proven (in the situation described in this question, the charge is unresolved). Although the Canadian Code of Ethics does not explicitly address this situation, Principle I.13 states that psychologists “abide by due process or other natural justice procedures for employment.”

The correct answer is: has acted unethically by violating the requirements of the ethical guidelines regarding employment procedures.

How well did you know this?
1
Not at all
2
3
4
5
Perfectly
71
Q

A graduate student’s dissertation chair, Dr. X, turns out to be of little help, and the student ends up obtaining advice and assistance from an associate professor, Dr. Z. Dr. X does review the dissertation when it is completed and she is listed as the dissertation chair. Several months later, the student decides to write an article for a professional journal that is based on his dissertation research. In terms of publication credit:
Select one:

A.
Dr. Z should be listed as first author if her contribution was substantial.

B.
Dr. Z should be listed as second author if her contribution warrants it.

C.
Dr. X should be listed as second author since she was the students official dissertation chair.

D.
Dr. X and Dr. Z should both be listed as co-authors.

A

Publication credit is addressed in Standard 8.12 of the APA’s Ethics Code and Principle III.7 of the Canadian Code of Ethics.

a. Incorrect See explanation for response b.
b. CORRECT Of the responses given, this one is best because it addresses the need to consider Dr. Z’s contribution to the research project (and Dr. X’s lack of contribution). Ethical guidelines emphasize that it is a person’s contribution, not his or her position or status, that determines publication credit. Moreover, when a published study is based on a student’s dissertation, the student should ordinarily be listed as first author.
c. Incorrect See explanation for response b.
d. Incorrect See explanation for response b.

The correct answer is: Dr. Z should be listed as second author if her contribution warrants it.

How well did you know this?
1
Not at all
2
3
4
5
Perfectly
72
Q

Dr. Marcos Manzetti uses a “sliding scale” that is based on a client’s current income to set his fees for therapy. Dr. Manzetti’s practice is:
Select one:

A.
ethically acceptable since it serves the best interests of his clients.

B.
explicitly recommended in the ethical guidelines.

C.
ethically unacceptable since it treats clients inequitably.

D.
explicitly prohibited in the ethical guidelines.

A

Sliding scale fees are not explicitly addressed in the APA’s Ethics Code or the Canadian Code of Ethics.

a. CORRECT Sliding scale fees for psychological services are consistent with the “spirit” of ethical guidelines as long as they are fair and serve the best interests of the client.
b. Incorrect See explanation above.
c. Incorrect See explanation above.
d. Incorrect See explanation above.

The correct answer is: ethically acceptable since it serves the best interests of his clients.

How well did you know this?
1
Not at all
2
3
4
5
Perfectly
73
Q

When consulting with a colleague about a “therapeutic impasse” you are having with a therapy client:
Select one:

A.
it is always necessary to advise the client about the consultation.

B.
it is necessary to get a signed waiver of confidentiality from the client before discussing confidential information with the consultant.

C.
it is unnecessary to advise the client of the consultation as long as only information that is relevant to the impasse is discussed.

D.
it is unnecessary to advise the client of the consultation as long as the client’s identity is not revealed.

A

Consultation is addressed in Standard 4.06 of the APA’s Ethics Code and Principles I.38 and 45 of the Canadian Code of Ethics.

a. Incorrect This answer does not accurately describe ethical requirements related to consultation.
b. Incorrect Obtaining a waiver of confidentiality is not necessary as long as the client”s identity is not revealed.
c. Incorrect This answer does not accurately describe ethical requirements related to consultation.
d. CORRECT This answer most accurately describes a psychologist’s ethical obligation when consulting with a colleague about a therapy client.

The correct answer is: it is unnecessary to advise the client of the consultation as long as the client’s identity is not revealed.

How well did you know this?
1
Not at all
2
3
4
5
Perfectly
74
Q

In terms of ethical guidelines, providing pro bono services is:
Select one:

A.
recommended.

B.
required.

C.
discouraged.

D.
prohibited.

A

Pro bono services are professional services that are provided for no compensation for the public good.

a. CORRECT Pro bono services are not directly addressed by the APA’s Ethics Code or the Canadian Code of Ethics. However, providing services for no compensation to serve the public good is certainly ethical. For example, pro bono services are consistent with General Principle A of the Ethics Code which states that psychologists “strive to contribute a portion of their professional time for little or no compensation or personal advantage.”
b. Incorrect Although psychologists are encouraged to provide services for little or no compensation, they are not required to do so.
c. Incorrect See explanation for response a.
d. Incorrect See explanation for response a.

The correct answer is: recommended.

How well did you know this?
1
Not at all
2
3
4
5
Perfectly
75
Q

Dr. Isaac Ibraham, a licensed psychologist, wants to expand his private practice. When doing so, he should keep in mind that uninvited in-person solicitations for therapy clients are:
Select one:

A.
always ethical.

B.
always unethical.

C.
unethical only when they provide potential clients with misleading or inaccurate information.

D.
unethical when the solicited individuals are susceptible to exploitation or undue influence.

A

This issue is addressed by Standard 5.06 of the APA’s Ethics Code and Principle III.31 of the Canadian Code of Ethics.

a. Incorrect See explanation for response d.
b. Incorrect See explanation for response d.
c. Incorrect Providing individuals with inaccurate or misleading information would be unethical, but this answer states that solicitation is unethical only in this circumstance, which is not the case.
d. CORRECT Although uninvited in-person solicitation of therapy clients is not entirely prohibited by ethical guidelines, the guidelines limit the types of in-person solicitation that are acceptable. Standard 5.06 states that “psychologists do not engage … in uninvited in-person solicitation of business [from persons who] … are vulnerable to undue influence”; and Principle III.31 states that psychologists do not exploit others by, for example, “taking advantage of trust or dependency to frighten clients into receiving services,” which might occur when in-person solicitations are uninvited.

The correct answer is: unethical when the solicited individuals are susceptible to exploitation or undue influence.

How well did you know this?
1
Not at all
2
3
4
5
Perfectly
76
Q

A student participant in a research study involving matched pairs decides to withdraw from the study. The chief investigator (who is a licensed psychologist) stresses to the student the importance of the student’s data to the study, but he insists that his data be withdrawn. The investigator should:
Select one:

A.
warn the student that withdrawal from the study will affect his course grade.

B.
remind the student that he signed an informed consent and cannot now request that his data be withdrawn.

C.
tell the student that his name will be removed from all data and include his data in the study.

D.
allow the student to withdraw from the study.

A

This issue is addressed by Standard 8.02 of the APA’s Ethics Code and Principle I.24 of the Canadian Code of Ethics.

a. Incorrect Simply warning the student of the negative consequences of withdrawing from the study would be coercive and, therefore, a violation of ethical guidelines.
b. Incorrect See explanation for response d.
c. Incorrect See explanation for response d.
d. CORRECT This answer is most consistent with ethical guidelines. Standard 8.02(a) states that research participants must be free to “decline to participate and to withdraw from the research once participation has begun”; and Principle I.24 similarly states that research participants must have “the option to refuse or withdraw at any time, without prejudice.”

The correct answer is: allow the student to withdraw from the study.

How well did you know this?
1
Not at all
2
3
4
5
Perfectly
77
Q

Dr. Pavel Petrovich, a licensed psychologist, has been seeing a client in therapy for over fourteen months with no apparent change in the client’s symptoms. Dr. Petrovich should:
Select one:

A.
advise the client that he is ethically obligated to terminate therapy.

B.
raise for discussion the possibility of termination and referral to another therapist.

C.
search the literature for discussions of similar cases that have been treated and reported by others.

D.
continue seeing the client since fourteen months is not an unusually long time for therapy to continue without a significant change in symptoms.

A

Termination of therapy is addressed in Standard 10.10 of the APA’s Ethics Code and Principle II.37 of the Canadian Code of Ethics.

a. Incorrect See explanation for response b.
b. CORRECT The action described in this answer is most consistent with ethical requirements. Standard 10.10(a), for example, states that “psychologists terminate therapy when it becomes reasonably clear that the client/patient no longer needs the service, is not likely to benefit, or is being harmed by continued service,” and Standard 10.10(c) states that, prior to termination, psychologists “suggest alternative service providers as appropriate.”
c. Incorrect See explanation for response b.
d. Incorrect In many cases, fourteen months is sufficiently long to consider the possibility of termination when a client has shown “no apparent change” in symptoms.

The correct answer is: raise for discussion the possibility of termination and referral to another therapist.

How well did you know this?
1
Not at all
2
3
4
5
Perfectly
78
Q

The Buckley Amendment establishes:
Select one:

A.
the right of parents or legal guardians of a child to inspect the childs school records.

B.
the right of patients to have access to their own hospital records.

C.
the requirement for employers to make reasonable modifications to tests for applicants with disabilities.

D.
the requirement for schools to provide a free appropriate public education to students with disabilities.

A

Knowing that the Buckley Amendment is also known as the Family Education Rights and Privacy Act (FERPA) would have helped you identify the correct answer to this question. Additional information about the Buckley Amendment is provided in the Ethics Manual.

a. CORRECT The Buckley Amendment states that any school district may be denied federal funds if parents or other legal guardians of students or students who have reached the age of majority are not given access to the students’ school records.
b. Incorrect See explanation for response a.
c. Incorrect See explanation for response a.
d. Incorrect See explanation for response a.

The correct answer is: the right of parents or legal guardians of a child to inspect the childs school records.

How well did you know this?
1
Not at all
2
3
4
5
Perfectly
79
Q

Dr. Carl Carlson is a licensed psychologist and professor in the psychology department at a large university. He is approached by a graduate student, Marla M., age 36, who asks him if he’d like to attend a baseball game on Friday night. Dr. Carlson is divorced and finds Marla to be a very attractive woman. In terms of ethical guidelines, if Dr. Carlson accepts Marla’s invitation, he will be:
Select one:

A.
clearly violating ethical guidelines.

B.
acting ethically since he is not the one who initiated a potential relationship.

C.
acting ethically as long as Marla is a student in a department other than the psychology department and will not be taking any classes in the psychology department.

D.
acting ethically as long as he does not become sexually involved with Marla until she is no longer affiliated with the university.

A

Dating students is ordinarily prohibited or discouraged because doing so represents a multiple relationship.

a. Incorrect Dr. Carlson may or may not be violating ethical guidelines, depending on the circumstances (see response c).
b. Incorrect The fact that Marla initiated the potential relationship is not relevant.
c. CORRECT If Marla is a graduate student in another department, a relationship with her would not represent a multiple relationship because Dr. Carlson is not likely to be her instructor or advisor.
d. Incorrect As long as Dr. Carlson does not have “supervisory, evaluative, or other authority” over Marla, it would not be a violation of ethical guidelines if he becomes sexually involved with her (see Standards 3.08 and 7.07 of the APA’s Ethics Code and Principle II.28 of the Canadian Code of Ethics). (Note that personal relationships with students might be prohibited by university policy but that this question is asking about ethical guidelines.)

The correct answer is: acting ethically as long as Marla is a student in a department other than the psychology department and will not be taking any classes in the psychology department.

How well did you know this?
1
Not at all
2
3
4
5
Perfectly
80
Q

A couple brings their ten-year-old son to therapy at the suggestion of his school counselor who is concerned because the boy has started arguing with his teacher and getting into physical fights with his classmates. After several sessions with the boy and his family, you receive a letter from the school principal requesting information about the boy’s condition, and a signed release from the boy’s parents is included with the letter. As an ethical psychologist, you should:
Select one:

A.
refuse to release the records to anyone but a licensed psychologist.

B.
refuse to release the records to anyone but the boy’s parents.

C.
release only that information you believe to be relevant to the school’s concerns.

D.
release photocopies of your complete file on the boy and his family.

A

Since the boy’s parents have signed a release, you would ordinarily provide information to the school principal - but not necessarily all of the information that has been requested.

a. Incorrect See explanation for response c.
b. Incorrect See explanation for response c.
c. CORRECT The most ethical course of action in this situation would be to release only information that is relevant to the school’s role in the boy”s treatment. Doing so would be consistent with the provisions of Standard 4.04(a) of the APA’s Ethics Code, which states that written and oral reports include “only information germane to the purpose for which the communication is made.” It is also consistent with the provisions of Principles I.37 and 39 of the Canadian Code of Ethics.
d. Incorrect See explanation for response c.

The correct answer is: release only that information you believe to be relevant to the school’s concerns.

How well did you know this?
1
Not at all
2
3
4
5
Perfectly
81
Q
Students in an introductory psychology class are required to participate in one of the university's ongoing research projects. This practice is:
Select one:

A.
ethical as long as students can choose to participate in an alternative activity instead.

B.
ethical as long as students are free to choose the research project they participate in.

C.
ethical as long as informed consents are obtained from students before they participate.

D.
clearly unethical.

A

This issue is addressed by Standard 8.04 of the APA’s Ethics Code and Principle I.36 of the Canadian Code of Ethics.

a. CORRECT This answer is most consistent with ethical guidelines. Standard 8.04(b), for example, states that “when research participation is a course requirement or an opportunity for extra credit, the prospective participant is given the choice of equitable alternative activities.”
b. Incorrect See explanation for response a.
c. Incorrect See explanation for response a.
d. Incorrect See explanation for response a.

The correct answer is: ethical as long as students can choose to participate in an alternative activity instead.

How well did you know this?
1
Not at all
2
3
4
5
Perfectly
82
Q

The 1976 Tarasoff decision established:
Select one:

A.
the right of a parent to inspect and request modifications to his/her childs school records.

B.
a psychologists duty to protect the intended victim of a therapy client.

C.
the 80% rule as a standard for determining the fairness of a selection test or other employment procedure.

D.
a psychologists duty to report suspected or known cases of child abuse.

A

The Tarasoff decision established that, under certain circumstances, client confidentiality must be breached when the client is a danger to others. Additional information on the ruling is provided in the Ethics Manual.

a. Incorrect See explanation for response b.
b. CORRECT Although the original 1974 Tarasoff decision established a “duty to warn” the intended victim of a therapy client, that decision was superceded by the 1976 rehearing of the case which established a “duty to protect” the intended victim. Specifically, the 1976 ruling states that a psychotherapist has a duty to protect an intended victim of a therapy client by warning “the intended victim or others likely to apprise the victim of the danger, … [notifying] the police, or … [taking] whatever other steps are reasonably necessary under the circumstances.”
c. Incorrect See explanation for response b.
d. Incorrect See explanation for response b.

The correct answer is: a psychologists duty to protect the intended victim of a therapy client.

How well did you know this?
1
Not at all
2
3
4
5
Perfectly
83
Q

When a psychologist learns that a colleague has committed an ethical violation, the psychologist:
Select one:

A.
may choose to ignore the violation if it is unlikely to be repeated by the colleague.

B.
must report the violation to the Ethics Committee or state/provincial licensing board.

C.
may attempt to resolve the situation informally by discussing it with the colleague if it seems appropriate to do so.

D.
should contact the colleague’s client to ascertain that a violation has actually occurred before taking any further action.

A

This issue is addressed in Standards 1.04 and 1.05 of the APA’s Ethics Code and Principles II.41 and II.42 of the Canadian Code of Ethics.

a. Incorrect See explanation for response c.
b. Incorrect See explanation for response c.
c. CORRECT Ethical guidelines require psychologists to take action when they learn that a colleague has committed an ethical violation. This may involve addressing the issue informally when it seems appropriate to do so or reporting the violation to the Ethics Committee or licensing board when it does not seem appropriate for informal resolution or has not been resolved adequately through informal resolution. (Note that answer b is not the best answer because it states that a psychologist MUST report the violation to the Ethics Committee or licensing board, which is not true.)
d. Incorrect See explanation for response c.

The correct answer is: may attempt to resolve the situation informally by discussing it with the colleague if it seems appropriate to do so.

How well did you know this?
1
Not at all
2
3
4
5
Perfectly
84
Q

The Ethics Committee is investigating a complaint against a psychologist and requests, by letter, that you provide the Committee with information about the complainant who is a former client of yours. When you check your records, you realize that you stopped seeing this client nine years ago. As an ethical psychologist, you should:
Select one:

A.
respond immediately to the Committee’s request by supplying it with the information it has asked for.

B.
respond immediately to the Committee’s request but provide it only with the information you feel is relevant to the complaint.

C.
respond to the Committee’s request by informing it that the information you have about the client is obsolete.

D.
respond immediately to the Committee’s request but supply it with the requested information only after verifying that the client has signed a release of information.

A

Confidentiality is an important concern in any action that you take as a psychologist, and many ethical dilemmas will require consideration of this issue.

a. Incorrect See explanation for response d.
b. Incorrect See explanation for response d.
c. Incorrect If the information in the file is obsolete, it would be important to note this in any response to the Ethics Committee. However, this response does not address the issue of confidentiality and, therefore, is not the best answer.
d. CORRECT This is most consistent with the provisions of the ethical guidelines, which require psychologists to respond to inquiries from Ethics Committees but, at the same, to address issues related to confidentiality (see, e.g., Standard 1.06 of the APA’s Ethics Code).

The correct answer is: respond immediately to the Committee’s request but supply it with the requested information only after verifying that the client has signed a release of information.

How well did you know this?
1
Not at all
2
3
4
5
Perfectly
85
Q

The owner of a small advertising agency refers an employee of the agency to Dr. K., a clinical psychologist, for an evaluation. The owner suspects that the employee is too “thick-headed” to be a benefit to the company. The owner and Dr. K. agree, prior to the evaluation, that the psychologist will report back to the owner as to whether or not additional training might improve the employee’s job performance. Following several sessions with the employee, the psychologist prepares a report, which he gives to the owner. On the basis of the report and the employee’s previous work history, the owner decides to fire the employee. The employee is very upset with Dr. K. for not informing him about the purpose of the evaluation and files a complaint against Dr. K. with the Ethics Committee. The Committee is likely to:
Select one:

A.
dismiss the complaint since complaints can be filed only by the clients of psychologists (in this case, the owner) or by other psychologists.

B.
dismiss the complaint since, in this situation, the psychologist could not reveal the purpose of the evaluation to the employee in order to obtain accurate information.

C.
dismiss the complaint since the psychologist had acted appropriately by providing information only to his client, the owner, who was responsible for informing the employee of the purpose of the evaluation.

D.
find that the psychologist acted unethically by not informing the employee of the purpose of the evaluation.

A

Ethical Standard 3.07 states that “When psychologists agree to provide services to a person or entity at the request of a third party, psychologists attempt to clarify at the outset of the service the nature of the relationship with all individuals or organizations involved. This clarification includes the role of the psychologist (e.g., therapist, consultant, diagnostician, or expert witness), an identification of who is the client, the probable uses of the services provided or the information obtained, and the fact that there may be limits to confidentiality.”

a. Incorrect This is not true. There is no restriction regarding who can file a complaint against a psychologist (except that complaints generally cannot be filed anonymously).
b. Incorrect This is not consistent with the provisions of the Ethics Code.
c. Incorrect This is not consistent with the provisions of the Ethics Code.
d. CORRECT In an actual case similar to this one (APA, 1987), the Ethics Committee censured the psychologist for not clearly explaining the purpose of the evaluation to the employee.

The correct answer is: find that the psychologist acted unethically by not informing the employee of the purpose of the evaluation.

How well did you know this?
1
Not at all
2
3
4
5
Perfectly
86
Q

Research on job satisfaction suggests that it:
Select one:

A.
is a relatively stable trait and is minimally affected by job changes.

B.
is relatively stable within the same job but unstable when measured across different jobs.

C.
is relatively unstable and varies over time both within the same job and across different jobs.

D.
may be stable or unstable within and across jobs depending on the characteristics of the particular worker.

A

Overall, job satisfaction seems to be a stable trait.

a. CORRECT This best summarizes the results of research on job satisfaction. It appears that job satisfaction is directly related to the tendency toward positive or negative affect, which is a stable characteristic. People with negative affect tend to be dissatisfied with work; people with positive affect tend to be satisfied.
b. Incorrect See explanation for response a.
c. Incorrect See explanation for response a.
d. Incorrect See explanation for response a.

The correct answer is: is a relatively stable trait and is minimally affected by job changes.

How well did you know this?
1
Not at all
2
3
4
5
Perfectly
87
Q

A group’s performance on a(n) __________ task is limited by the performance of the least skilled or knowledgeable member of the group.
Select one:

A.
additive

B.
compensatory

C.
disjunctive

D.
conjunctive

A

Group tasks are categorized as additive, compensatory, disjunctive, or conjunctive. Additional information on these tasks is included in the Industrial-Organizational Psychology chapter of the written study materials.

a. Incorrect On additive tasks, the coordinated efforts of several people add together to form the group product.
b. Incorrect When performing a compensatory task, the average of the contributions of all members of the group represents the group product.
c. Incorrect On a disjunctive task, the solution of one member of the group is chosen by the group to be the group’s solution.
d. CORRECT On a conjunctive task, the performance of the least competent member places a limit on the group’s product or performance. Mountain climbing is an example of a conjunctive task.

The correct answer is: conjunctive

How well did you know this?
1
Not at all
2
3
4
5
Perfectly
88
Q

Expectancy theory predicts that worker motivation depends on several factors including “valence”, which refers to:
Select one:

A.
the employees beliefs about the value of rewards provided for successful performance.

B.
the employees beliefs about the inherent (intrinsic) value of the work itself.

C.
the strength of the worker’s motivation and organizational commitment.

D.
the strength of the effort (versus ability) component of the worker’s motivation.

A

According to expectancy theory, motivation is a function of three components: expectancy, instrumentality, and valence. For the exam, you want to be familiar with the characteristics of all three components (see the Industrial Organizational Psychology chapter of the written study materials).

a. CORRECT Valence refers to the value a worker places on the outcomes that will be provided for successful performance. When these outcomes have positive valence, this increases the worker’s motivation to perform.
b. Incorrect See explanation for response a.
c. Incorrect See explanation for response a.
d. Incorrect See explanation for response a.

The correct answer is: the employees beliefs about the value of rewards provided for successful performance.

How well did you know this?
1
Not at all
2
3
4
5
Perfectly
89
Q

According to Fiedler’s contingency model of leadership, high LPC leaders:
Select one:

A.
are always more effective than low LPC leaders.

B.
are more effective than low LPC leaders in moderately favorable situations.

C.
elicit less trust from supervisees than do low LPC leaders.

D.
elicit less intrinsic motivation from supervisees than do low LPC leaders.

A

Fiedler’s LPC (least preferred coworker) scale measures the extent to which a leader describes ineffective subordinates in positive terms. A high LPC leader is one who has nice things to say even about inefficient workers.

a. Incorrect Fiedler’s theory is a “contingency” theory and predicts that the most effective leadership style depends on the nature of the situation.
b. CORRECT According to Fiedler, low LPC leaders are most effective in low and high favorable situations, while high LPC leaders are most effective in moderately favorable situations.
c. Incorrect This is not predicted by Fiedler’s theory.
d. Incorrect Fiedler did not directly address the issue of job motivation but, because his theory is a contingency theory, it implies that the ability of high and low LPC leaders to foster the motivation of employees would depend on the nature of the situation.

The correct answer is: are more effective than low LPC leaders in moderately favorable situations.

How well did you know this?
1
Not at all
2
3
4
5
Perfectly
90
Q

An organizational psychologist finds that his newly developed selection test has different validity coefficients for male and female applicants and that the difference is statistically significant. This suggests that the test has:
Select one:

A.
incremental validity.

B.
discriminant validity.

C.
differential validity.

D.
convergent validity.

A

For exam, you want to be familiar with all of the terms listed in the answers to this question. Additional information about these terms is provided in the Industrial-Organizational Psychology chapter of the written study materials.

a. Incorrect A test’s incremental validity refers to the benefits that use of the test provides with regard to decision-making accuracy.
b. Incorrect A test has discriminant (divergent) validity when it has low correlations with tests that measure different traits, characteristics, or abilities. Discriminant validity provides evidence of a test”s construct validity.
c. CORRECT The selection test described in this question has different levels of validity for males and females – i.e., it has differential validity.
d. Incorrect A test has convergent validity when it correlates highly with tests that measure the same trait. Convergent validity provides evidence of construct validity.

The correct answer is: differential validity.

How well did you know this?
1
Not at all
2
3
4
5
Perfectly
91
Q

In organizations, the 80% rule is used to determine if:
Select one:

A.
a selection procedure is having an adverse impact.

B.
a selection procedure is cost effective.

C.
a performance appraisal measure has adequate relevance.

D.
a performance appraisal measure has adequate utility.

A

The 80% rule is described in the EEOC’s Uniform Guidelines on Employee Selection Procedures and is used to determine if a selection test or other employment procedure is discriminating against members of a group protected by law.

a. CORRECT As defined by the 80% rule, a selection test is having an adverse impact when the proportion of minority-group applicants who are selected is less than 80% of the proportion of majority-group applicants who are selected.
b. Incorrect See explanation above.
c. Incorrect See explanation above.
d. Incorrect See explanation above.

The correct answer is: a selection procedure is having an adverse impact.

92
Q

Lewin’s force field theory describes planned change in organizations as involving which of the following stages?
Select one:

A.
unfreezing, changing, and refreezing

B.
forming, performing, and re-forming

C.
identifying, planning, and acting

D.
setting the stage, problem-solving, and reaching an agreement

A

According to Lewin (1951), organizations continuously respond to forces that either promote or resist change.

a. CORRECT Unfreezing, changing, and refreezing are the three stages of planned change identified by Lewin.
b. Incorrect See explanation for response a.
c. Incorrect See explanation for response a.
d. Incorrect These are the three stages of mediation.

The correct answer is: unfreezing, changing, and refreezing

93
Q

The primary purpose of a realistic job preview is to:
Select one:

A.
increase the number of applicants for a job.

B.
develop an accurate job description.

C.
reduce on-the-job conflicts between managers and employees.

D.
reduce employee dissatisfaction and turnover.

A

Realistic job previews may consist of job descriptions, videos, discussions with current employees, work samples, and/or other procedures or techniques.

a. Incorrect See explanation for response d.
b. Incorrect See explanation for response d.
c. Incorrect See explanation for response d.
d. CORRECT Two assumptions underlying the use of realistic job previews are that turnover is related to unrealistic expectations about the job and that providing applicants with accurate information about what the job entails will reduce unrealistic expectations and thereby decrease dissatisfaction and turnover.

The correct answer is: reduce employee dissatisfaction and turnover.

94
Q

Donald Super’s _____________ graphically depicts the correspondence between an individual’s life roles and life stages.
Select one:

A.
life career rainbow

B.
life staircase

C.
mandala

D.
octagon

A

Super uses several pictures to illustrate elements of his life-space, life-span theory of career development.

a. CORRECT The life career rainbow relates the individual’s nine life roles (e.g., child, student, citizen) to five life stages (growth, exploration, establishment, maintenance, and disengagement).
b. Incorrect Super does not use a life staircase to depict components of his theory.
c. Incorrect Super does not use a mandala to depict his theory.
d. Incorrect Super does not use an octagon to illustrate elements of his theory.

The correct answer is: life career rainbow

95
Q

A transformational leader uses “framing” in order to:
Select one:

A.
make the organization’s goals more meaningful to employees.

B.
clarify the consequences of undesirable performance.

C.
reduce personal biases in decision-making.

D.
adapt his/her leadership style to the characteristics of subordinates.

A

Even if you’re unfamiliar with “framing,” you may have been able to identify the correct answer with your knowledge of transformational leadership.

a. CORRECT Transformational leaders recognize the need for change and are able to communicate that need to employees. Framing is one technique these leaders use to inspire employees and involves describing the organization’s goals in a way that makes them more meaningful to employees.
b. Incorrect See the explanation for response a.
c. Incorrect The term “framing” is also used to refer to a type of decision-making bias. In that context, framing occurs when the description of a problem in a positive or negative way influences the type of decision that is made. However, this question is asking about the use of framing by transformational leaders.
d. Incorrect See the explanation for response a.

The correct answer is: make the organization’s goals more meaningful to employees.

96
Q

The best conclusion that can be drawn about the four-day (compressed) workweek is that it has:
Select one:

A.
long-lasting positive effects on attitudes and productivity.

B.
long-lasting negative effects on attitudes and productivity.

C.
positive effects on attitudes but little or no impact on productivity.

D.
positive effects on attitudes initially but, over time, increasingly negative effects on both attitudes and productivity.

A

The compressed workweek requires employees to work a 4-day, 40-hourweek.

a. Incorrect See explanation for response c.
b. Incorrect See explanation for response c.
c. CORRECT Overall, alternative work schedules have been found to have more beneficial effects on attitudes than on productivity and this seems to be particularly true for the compressed workweek. For example, a meta-analysis of the research on the compressed workweek by Baltes and his colleagues (1999) found an effect size of .59 for satisfaction and .04 for productivity.
d. Incorrect See explanation for response c.

The correct answer is: positive effects on attitudes but little or no impact on productivity.

97
Q

According to Hersey and Blanchard’s situational leadership model, “employee maturity” is a function of the employee’s:
Select one:

A.
stage of career development.

B.
personality.

C.
skills and willingness to assume responsibility.

D.
beliefs about the meaning and value of work.

A

Hersey and Blanchard propose that a leader is most effective when his/her behavior matches the employee”s level of maturity.

a. Incorrect See explanation for response c.
b. Incorrect See explanation for response c.
c. CORRECT Hersey and Blanchard distinguish between two aspects of employee maturity: Job maturity refers to the employee’s knowledge and skills, while psychological maturity refers to the employee’s self-confidence and willingness to assume responsibility.
d. Incorrect See explanation for response c.

The correct answer is: skills and willingness to assume responsibility.

98
Q

Assessment centers are most commonly used to:
Select one:

A.
hire and promote clerical workers.

B.
hire and promote managers.

C.
train semi-skilled and skilled workers.

D.
train salespeople.

A

Assessment centers were originally used during World War II as a means of selecting OSS agents. They are now used primarily for evaluating and predicting the performance of managerial-level employees.

a. Incorrect See explanation for response b.
b. CORRECT Assessment centers are used to evaluate managerial-level employees for the purpose of selection, promotion, or training and involve having participants engage in a variety of exercises, many of which simulate the actual tasks a manager performs on-the-job.
c. Incorrect See explanation for response b.
d. Incorrect See explanation for response b.

The correct answer is: hire and promote managers.

99
Q

In organizations, the “paired comparison” technique is used to:
Select one:

A.
evaluate an employee’s job performance.

B.
make hiring and placement decisions.

C.
identify an employee’s training needs.

D.
determine the appropriate compensation for a job.

A

Answer A is correct. The paired comparison technique is a method of performance appraisal. See the Industrial-Organizational Psychology chapter for a description of other specific methods that you’re likely to encounter on the exam. As its name implies, the use of the paired comparison technique involves comparing each employee to every other employee in pairs on each dimension of job performance.

The correct answer is: evaluate an employee’s job performance.

100
Q

The rational-economic model of decision-making is based on the assumption that:
Select one:

A.
decision-makers place more emphasis on the costs of certain decisions than on other consequences of their decisions.

B.
decision-makers have complete information about all alternatives and their consequences before making decisions.

C.
individuals are better than groups at making decisions, especially under stressful conditions.

D.
decision-makers knowledge about possible alternatives is always incomplete.

A

As its name implies, the rational-economic model assumes that decisions are based on a rational process.

a. Incorrect This is not an assumption of the rational-economic model.
b. CORRECT From the perspective of the rational-economic model, “rational” means considering all alternatives and their consequences before making a decision.
c. Incorrect This is not an assumption of this model.
d. Incorrect A criticism of the rational-economic model is that it does not take into account that a decision-maker’s knowledge about alternatives is often incomplete.

The correct answer is: decision-makers have complete information about all alternatives and their consequences before making decisions.

101
Q

The primary purpose of a(n) ____________ is to obtain detailed information about job requirements in order to facilitate decisions related to compensation.
Select one:

A.
needs assessment

B.
organizational analysis

C.
job analysis

D.
job evaluation

A

Of the procedures listed in the answers, only one is conducted specifically to facilitate decisions related to compensation.

a. Incorrect A needs assessment is conducted to identify training needs.
b. Incorrect An organizational analysis is often part of a needs assessment and is conducted to identify the organization’s goals.
c. Incorrect A job analysis is conducted to identify the essential characteristics of a job. Job analysis serves several functions in an organization and may be the first step in a job evaluation.
d. CORRECT A job evaluation is conducted specifically for the purpose of determining the relative worth of jobs in order to set wages and salaries.

The correct answer is: job evaluation

102
Q

An organizational psychologist would most likely recommend the use of vestibule training for which of the following jobs?
Select one:

A.
airline pilot

B.
word processor

C.
salesperson

D.
clinical psychologist

A

Vestibule training involves the use of a simulated job environment.

a. CORRECT Vestibule training is used when on-the-job training would be too costly or hazardous, as it would be in pilot training.
b. Incorrect See explanation for response a.
c. Incorrect See explanation for response a.
d. Incorrect See explanation for response a.

The correct answer is: airline pilot

103
Q

If you were hired by a large company to develop a new training program, your first step would probably be to conduct a:
Select one:

A.
needs analysis.

B.
job evaluation.

C.
summative evaluation.

D.
formative evaluation.

A

For the exam, you want to be able to distinguish between needs analysis, job analysis, and job evaluation. All three are described in the Industrial-Organizational Psychology chapter of the written study materials.

a. CORRECT Training program development begins with a needs analysis (also known as a needs assessment), which usually consists of three components: an organizational analysis, a job analysis, and a person analysis.
b. Incorrect A job evaluation is conducted to determine the appropriate compensation for a job.
c. Incorrect A summative evaluation is conducted to determine the effects of a training program or intervention after it has been developed.
d. Incorrect A formative evaluation is conducted while a training program or intervention is being developed to determine if modifications are required in order for the program or intervention to achieve its goals.

The correct answer is: needs analysis.

104
Q

A manager who adheres to the principles of “scientific management” would most likely agree with which of the following?
Select one:

A.
Employees view work as natural as play and actively seek autonomy and responsibility.

B.
An employee’s relationships with his/her coworkers are important determinants of job satisfaction.

C.
Because employees value money more than other incentives, pay is the most effective motivator.

D.
An employee’s unfulfilled needs take precedence over other needs, and rewards should be determined accordingly.

A

Scientific management was developed by Frederick Taylor (1911) who applied the scientific method to the study of job productivity

a. Incorrect This describes McGregor’s Theory Y managers.
b. Incorrect This is an assumption of the human relations movement, which was an alternative to scientific management.
c. CORRECT Taylor believed that employees are motivated primarily by economic self-interest and, therefore, that money is the most effective motivator.
d. Incorrect This sounds like Maslow’s need hierarchy theory.

The correct answer is: Because employees value money more than other incentives, pay is the most effective motivator.

105
Q

In the context of Holland’s RIASEC model, ___________ refers to the difference in the magnitude of scores an examinee obtains on the six personality types.
Select one:

A.
consistency

B.
differentiation

C.
identity

D.
congruence

A

All of the terms listed in the answers to this question refer to constructs that are important in Holland’s theory.

a. Incorrect In Holland’s model, consistency refers to the similarity of the individual’s primary interests (highest codes). For example, realistic and enterprising are consistent but realistic and social are not consistent.
b. CORRECT The question provides an accurate description of differentiation. A highly differentiated person obtains a high score on one personality type and low scores on all other types.
c. Incorrect Vocational identity refers to the degree to which the individual has a clear and stable picture of his or her goals, interests, and talents.
d. Incorrect Congruence refers to the degree of similarity between the person and the person’s work environment.

The correct answer is: differentiation

106
Q

The highest levels of learning and performance are usually associated with:
Select one:

A.
low levels of arousal.

B.
moderate levels of arousal.

C.
high levels of arousal.

D.
variability in levels of arousal.

A

Even if you’re not familiar with the Yerkes-Dodson law (which proposes that learning and performance are curvilinearly related to arousal), you may have been able to identify the correct answer on the basis of common sense - i.e., most people perform best when their level of arousal is somewhere between “none” and “a lot.”

a. Incorrect A low level of arousal can lead to boredom which can negatively affect learning and performance.
b. CORRECT Moderate levels of arousal are associated with the highest levels of learning and performance. This is particularly true when moderate levels of arousal are combined with moderate task difficulty.
c. Incorrect A high level of arousal can lead to stress and fatigue, which can negatively affect learning and performance.
d. Incorrect Varying levels of arousal would not be optimal for learning and performance.

The correct answer is: moderate levels of arousal.

107
Q

Of the following, which is the best way to control the halo effect?
Select one:

A.
Train the raters.

B.
Use peer ratings.

C.
Make the results of assessments available to employees.

D.
Involve raters in the development of the rating scale.

A

The assessment of employee performance often involves subjective measures, which are associated with rater biases. One bias is the halo effect, which occurs when a rater’s rating of an employee on one dimension of job performance affects his/her rating of the employee on other dimensions.

a. CORRECT Training raters in the use of rating scales increases the accuracy of ratings, and research has demonstrated that comprehensive training can significantly reduce the halo effect. The halo effect and other rater biases can also be reduced with the use of multiple raters, ratings based on specific rather than global behaviors, and the use of certain types of scales, such as forced-choice and paired comparison scales.
b. Incorrect Peer ratings, although they have been found to be good predictors of training success and subsequent promotions, have not been found to be an effective method of reducing the halo effect.
c. Incorrect Making such results available to employees enhances employee acceptance of the appraisal process, but it does not help to reduce the halo effect.
d. Incorrect This does not represent a direct method of decreasing the halo effect, although rater involvement in the entire rating process can increase a rater’s motivation to fill out rating scales accurately and thoroughly.

The correct answer is: Train the raters.

108
Q

Which of the following would be LEAST useful for eliminating the halo effect, the central tendency bias, and other rater biases from performance ratings?
Select one:

A.
paired comparison

B.
Likert scale

C.
forced distribution

D.
ranking

A

A major problem with subjective rating scales is that they are susceptible to rater biases and thus may not provide accurate information about an employee’s job performance. However, biases can be controlled by using certain types of subjective rating scales.

a. Incorrect The paired comparison method requires the rater to compare each employee with each other employee and, therefore, does eliminate response biases.
b. CORRECT Likert-type scales and other graphic rating scales are very susceptible to rater biases, especially when the dimensions of job behavior being rated are very global.
c. Incorrect The forced distribution is a method of controlling rater biases.
d. Incorrect Like the paired comparison and forced distribution methods, ranking is a relative technique and helps reduce rating biases.

The correct answer is: Likert scale

109
Q

An employer can refuse to hire older workers:
Select one:

A.
only when there is a much larger proportion of younger than older applicants for the job.

B.
when previous experience with an older worker has suggested that they cannot perform the job successfully.

C.
when the employer hires older workers for other jobs in the company.

D.
when younger age is clearly required by the nature of the job.

A

Title VII and related legislation provides for exceptions to antidiscrimination requirements when discrimination is necessary for the normal operation of the business.

a. Incorrect This is not an acceptable reason for not hiring older workers.
b. Incorrect One or a few “bad experiences” would not be sufficient justification for not hiring older workers.
c. Incorrect This is not considered an adequate remedy by antidiscrimination legislation.
d. CORRECT Age discrimination is acceptable if age is clearly a “BFOQ” (bona fide occupational qualification). For instance, Disneyland would probably not get in trouble with the EEOC for hiring only younger men for the position of “Prince Charming.”

The correct answer is: when younger age is clearly required by the nature of the job.

110
Q

Implosive therapy and flooding share in common which of the following?
Select one:

A.
They both involve exposure to an unconditioned (aversive) stimulus.

B.
They are both based on counterconditioning.

C.
They both involve the use of negative reinforcement.

D.
They both lead to extinction of the undesirable response.

A

Implosive therapy and flooding are behavioral treatments based on the principles of classical conditioning.

a. Incorrect Exposure is to the conditioned, not the unconditioned, stimulus.
b. Incorrect Systematic desensitization utilizes counterconditioning.
c. Incorrect As noted above, both techniques are derived from classical (not operant)conditioning.
d. CORRECT Through exposure to the feared stimulus either in vivo (flooding) or in imagination (implosive therapy), the conditioned response is eventually extinguished.

The correct answer is: They both lead to extinction of the undesirable response.

111
Q

Bandura’s social learning theory predicts that:
Select one:

A.
cognitive events act as intervening variables in the acquisition of a new behavior.

B.
social approval is a powerful primary reinforcement.

C.
reinforcement has a greater impact on learning than on performance.

D.
the acquisition of new behaviors is the result of internalized S-R connections.

A

According to social learning theory, an individual learn (acquire) a new behavior simply by observing another person engage in that behavior.

a. CORRECT Bandura concluded that observational learning involves four processes: attention, retention, reproduction, and motivation. The first two of these, attention (attending to and accurately perceiving the behavior), and retention (symbolic processing of the modeled behavior) are cognitive processes.
b. Incorrect This is not an assumption of Bandura’s social learning theory.
c. Incorrect This is the opposite of what is true. Bandura has proposed that reinforcement is more important for performance than learning.
d. Incorrect This is not an assumption of Bandura’s social learning theory.

The correct answer is: cognitive events act as intervening variables in the acquisition of a new behavior.

112
Q

According to Albert Ellis, maladaptive behavior is caused primarily by which of the following?
Select one:

A.
learned helplessness

B.
boundary disturbances

C.
a lack of response-contingent reinforcement

D.
irrational thoughts and beliefs

A

Knowing that Ellis’s approach is classified as a type of cognitive-behavioral therapy would have helped you identify the correct response to this question.

a. Incorrect See explanation for response d.
b. Incorrect See explanation for response d.
c. Incorrect See explanation for response d.
d. CORRECT The basic premise of Ellis’s rational-emotive behavior therapy (REBT) is that maladaptive behaviors and emotions are the result of irrational thoughts and beliefs. Thus, the primary goal of REBT is to replace the client”s irrational thoughts and beliefs with more rational, adaptive ones.

The correct answer is: irrational thoughts and beliefs

113
Q

Elmo E., age 5, has learned that, if he has a tantrum whenever his parents want him to eat food he doesn’t like at mealtime, his parents will let him eat something else instead in order to get Elmo to stop crying. In this situation, the parents’ behavior (letting Elmo eat something other than the disliked food) is being maintained by which of the following?
Select one:

A.
escape conditioning

B.
avoidance conditioning

C.
stimulus discrimination

D.
stimulus generalization

A

In this situation, the parents’ behavior (letting Elmo eat something other than the disliked food) results in termination of Elmo’s tantrum. In other words, the parents’ behavior is being negatively reinforced.

a. CORRECT Escape conditioning occurs when a person”s behavior is maintained because it allows the person to escape an undesirable stimulus. Escape conditioning is a type of negative reinforcement.
b. Incorrect In avoidance conditioning, a cue signals that the undesirable stimulus is about to begin. In this situation, the individual can avoid the stimulus entirely by performing the appropriate behavior as soon as he/she perceives the cue. Avoidance conditioning is another type of negative reinforcement. (For a more detailed description of escape and avoidance conditioning, see the Learning Theory chapter of the written study materials.)
c. Incorrect In stimulus discrimination, a cue (stimulus) signals whether or not a reinforcer will be provided when the individual performs a particular behavior.
d. Incorrect In stimulus generalization, the individual responds with a CR to stimuli that are similar to the original CS.

The correct answer is: escape conditioning

114
Q

In Pavlov’s research, experimental animals began to salivate in response to a tone after the tone had been presented repeatedly with meat powder. In this situation, the tone was:
Select one:

A.
a conditioned stimulus.

B.
an unconditioned stimulus.

C.
a primary reinforcer.

D.
a secondary reinforcer.

A

In classical conditioning, a conditioned stimulus (CS) elicits a conditioned response (CR) as a result of its pairing with an unconditioned stimulus (US) that naturally elicits the unconditioned response (UR). If this is confusing or difficult to remember, the illustration of the classical conditioning process in the Learning Theory chapter of the written materials may help clarify it.

a. CORRECT In Pavlov’s studies, the tone was a conditioned stimulus (CS) - i.e., it elicited salivation only as the result of being paired with an unconditioned stimulus (meat powder) that naturally elicited salivation.
b. Incorrect An unconditioned stimulus (US) elicits a response without conditioning (learning trials). In Pavlov”s studies, the unconditioned stimulus was the meat powder.
c. Incorrect In operant conditioning, primary reinforcers are inherently reinforcing. Food and water are examples of primary reinforcers.
d. Incorrect In operant conditioning, secondary reinforcers are not inherently reinforcing but acquire their reinforcing value by being associated with primary reinforcers. Tokens in a token economy are secondary reinforcers because they can be exchanged for primary reinforcers.

The correct answer is: a conditioned stimulus.

115
Q

Samuel S., age 43, says, “Even though I’ve just been given a bonus at work, I feel like I’m about to be fired.” As defined by Aaron Beck, Samuel is exhibiting which of the following cognitive distortions?
Select one:

A.
mustabatory thinking

B.
polarized thinking

C.
personalization

D.
arbitrary inference

A

The phenomena listed in the answers to this question are all cognitive distortions that have been identified as contributors to maladaptive behavior.

a. Incorrect “Mustabatory thinking” is a term used by Ellis and refers to the belief that certain conditions must be met - e.g., “I must be liked by everyone; if not, I’m a terrible person.”
b. Incorrect Polarized (dichotomous) thinking involves seeing things as either “black or white.”
c. Incorrect Pesonalization involves mistakenly viewing oneself as the source of an event that actually had another cause.
d. CORRECT As defined by Beck, arbitrary inference involves drawing a conclusion that is made in the absence of supporting evidence or in the presence of contradictory evidence.

The correct answer is: arbitrary inference

116
Q

Research on the serial position effect provides support for the distinction between:
Select one:

A.
iconic and echoic memory.

B.
distributed and parallel processing.

C.
short- and long-term memory.

D.
proactive and retroactive interference.

A

The serial position effect refers to the tendency to recall items in the beginning and end of a list better than items in the middle of the list, especially when recall occurs immediately after rehearsing the list.

a. Incorrect See explanation for response c.
b. Incorrect See explanation for response c.
c. CORRECT One explanation for the serial position effect is that items in the beginning of the list have been stored in long-term memory, while those in the end of the list are still present in short-term memory.
d. Incorrect See explanation for response c.

The correct answer is: short- and long-term memory.

117
Q

When using the Premack Principle to modify a behavior, the reinforcer is:
Select one:

A.
applied intermittently.

B.
a generalized conditioned reinforcer.

C.
a behavior that occurs frequently.

D.
a stimulus that naturally elicits the desired behavior.

A

The Premack Principle is a type of positive reinforcement.

a. Incorrect See explanation for response c.
b. Incorrect See explanation for response c.
c. CORRECT When using the Premack Principle, a high-frequency behavior is used as the reinforcer for a low-frequency behavior. The Premack Principle is also known as “Grandma”s Rule” - e.g., “you can go out and play once you”ve cleaned your room.”
d. Incorrect See explanation for response c.

The correct answer is: a behavior that occurs frequently.

118
Q

A behavior therapist working with a child who bites her nails sets a timer so that it rings every 5 minutes. The child is then given a reward for every 5-minute period that she doesn”t bite her nails but engages in alternative activities instead. The therapist is using which of the following techniques?
Select one:

A.
differential reinforcement

B.
response cost

C.
overcorrection

D.
Premack Principle

A

The child is being reinforced for every 5-minute period during which she engages in behaviors other than nailbiting.

a. CORRECT Differential reinforcement is what it sounds like - i.e., it involves reinforcing behaviors other than (different from) the target behavior.
b. Incorrect Response cost involves removing a specific reinforcer following a behavior in order to decrease the behavior.
c. Incorrect Overcorrection is used to reduce undesirable behaviors and involves having the individual engage in alternative behaviors after performing the undesirable behavior (i.e., returning the environment to its previous condition and/or practicing correct behaviors).
d. Incorrect When using the Premack Principle, a high-frequency behavior is used as a reinforcer for a low-frequency behavior in order to increase the occurrence of the low-frequency behavior.

The correct answer is: differential reinforcement

119
Q

John Watson found that, after Little Albert had been classically conditioned to respond with fear to a white rat, the child also responded with fear to other white furry objects. Little Albert’s reaction to other white furry objects was the result of:
Select one:

A.
higher-order conditioning.

B.
negative transfer.

C.
pseudoconditioning.

D.
stimulus generalization.

A

Little Albert’s response to a particular stimulus (the white rat) generalized to similar stimuli (other white furry objects).

a. Incorrect Higher-order conditioning is a classical conditioning technique in which a neutral stimulus elicits a conditioned response as a result of its pairing with a conditioned stimulus. In this situation, other white furry objects were not “paired with” the white rat.
b. Incorrect Negative transfer occurs when previous learning interferes with present learning.
c. Incorrect Pseudoconditioning occurs when a neutral stimulus elicits a response due to the accidental pairing of the stimulus with another stimulus that evokes that response or when repeated exposure to a US increases the likelihood that the individual will respond to a neutral stimulus with a response similar to the UR.
d. CORRECT Stimulus generalization occurs when an individual responds with a conditioned response to stimuli that are similar to the original conditioned stimulus.

The correct answer is: stimulus generalization.

120
Q

Research using the dismantling strategy has found that the effectiveness of systematic desensitization as a treatment for anxiety is attributable to which of the following?
Select one:

A.
escape conditioning

B.
extinction

C.
satiation

D.
higher-order conditioning

A

Systematic desensitization was developed as an application of reciprocal inhibition (counterconditioning). However, research using the dismantling strategy suggests that reciprocal inhibition is not the essential component.

a. Incorrect See explanation for response b.
b. CORRECT The results of research using the dismantling strategy has led researchers to conclude that repeated exposure to feared stimuli leads to extinction of the undesirable anxiety response and is responsible for the benefits of systematic desensitization. (For a description of the dismantling strategy, see the Learning Theory chapter of the written study materials.)
c. Incorrect See explanation for response b.
d. Incorrect See explanation for response b.

The correct answer is: extinction

121
Q

The technique known as stress inoculation consists of three overlapping phases. These are:
Select one:

A.
preparation, action, and reflection.

B.
engagement/motivation, behavior change, and generalization.

C.
initiation of treatment, primary treatment/rehabilitation, and aftercare.

D.
conceptualization/education, skill acquisition and rehearsal, and application.

A

Stress inoculation (Meichenbaum & Jaremko, 1982) was designed to help people deal with stressful events by increasing their coping skills.

a. Incorrect See explanation for response d.
b. Incorrect See explanation for response d.
c. Incorrect See explanation for response d.
d. CORRECT Stress inoculation is a cognitive-behavioral technique that consists of the three phases listed in this response – i.e., conceptualization/education (also known as the cognitive preparation phase), skills acquisition and rehearsal, and application.

The correct answer is: conceptualization/education, skill acquisition and rehearsal, and application.

122
Q

When using a continuous schedule of reinforcement to increase the occurrence of a desirable behavior, you would be most concerned about which of the following?
Select one:

A.
satiation

B.
sensitization

C.
experimental neurosis

D.
spontaneous recovery

A

A continuous schedule of reinforcement involves reinforcing the individual each time he/she emits the target response.

a. CORRECT Satiation occurs when a reinforcer loses its reinforcing value because the organism has become satisfied (satiated). It is a potential problem when using positive reinforcement, especially when the reinforcer is a primary reinforcer and is delivered on a continuous schedule.
b. Incorrect In the context of learning theory, sensitization has at least two meanings: It refers to either the initial increase in responsiveness to a stimulus when it is presented repeatedly or to the increase in responsiveness to a stimulus that occurs after exposure to a very intense or irritating stimulus.
c. Incorrect Experimental neurosis occurs in classical conditioning when the organism is required to discriminate between very similar stimuli.
d. Incorrect Spontaneous recovery refers to the return of a conditioned response following the apparent extinction of that response.

The correct answer is: satiation

123
Q

Which of the following best describes elaborative rehearsal?
Select one:

A.
making new information meaningful

B.
consciously repeating new information

C.
practicing a new skill past the point of mastery

D.
thinking aloud while studying

A

Researchers interested in memory distinguish between two types of rehearsal - elaborative and maintenance.

a. CORRECT Elaborative rehearsal involves making new information meaningful in order to enhance its retention and retrieval. Relating new information to previously acquired information is one type of elaborative rehearsal.
b. Incorrect This sounds more like maintenance rehearsal which involves the rote repetition of information.
c. Incorrect This describes “overlearning.”
d. Incorrect This does not describe elaborative rehearsal.

The correct answer is: making new information meaningful

124
Q

The law of effect was proposed by:
Select one:

A.
Tolman.

B.
Thorndike.

C.
Watson.

D.
Bandura.

A

According to the law of effect, a response that is followed by a “satisfying state of affairs” is more likely to be repeated.

a. Incorrect Tolman is associated with cognitive maps.
b. CORRECT Thorndike’s research established a connection between the consequences of a behavior and the likelihood that the behavior will be repeated, and he referred to this connection as the law of effect.
c. Incorrect Watson used Pavlovian (classical) conditioning to explain the development of fear responses.
d. Incorrect Bandura is most associated with social (observational) learning theory.

The correct answer is: Thorndike.

125
Q

Children living in a residential facility are rewarded with a specific number of tokens each time they perform desirable behaviors but must return a certain number of tokens when they engage in undesirable behaviors. The latter technique (returning tokens) exemplifies which of the following?
Select one:

A.
negative reinforcement

B.
response cost

C.
Premack Principle

D.
differential reinforcement

A

This question is asking which technique involves removing a stimulus following a behavior in order to reduce or eliminate that behavior. Note that, for the exam, you want to be familiar wih the four techniques listed in the answers to this question. These techniques are all described in greater detail in the Learning Theory chapter of the written study materials.

a. Incorrect Negative reinforcement involves removing a stimulus following a behavior, but its goal is to increase the behavior, not to decrease it.
b. CORRECT Response cost is used to eliminate an undesirable behavior and involves imposing a penalty or fine (i.e., removing a specific stimulus) whenever the undesirable behavior occurs. Response cost is an application of negative punishment.
c. Incorrect The Premack Principle involves using a high frequency behavior as a reinforcer for a low frequency behavior in order to increase the low frequency behavior.
d. Incorrect Differential reinforcement is used to decrease an undesirable behavior by reinforcing alternative behaviors. It does not involve removing a specific stimulus (e.g., tokens) following the undesirable behavior, which is what is occurring in the situation described in this question.

The correct answer is: response cost

126
Q

The process of gradually removing a prompt until it is no longer provided is referred to as:
Select one:

A.
extinction.

B.
shaping.

C.
fading.

D.
thinning.

A

The process of gradually eliminating prompts is referred to as “fading.”

a. Incorrect Extinction refers to the removal of an unconditioned stimulus (classical conditioning) or a reinforcer (operant conditioning).
b. Incorrect Shaping involves reinforcing successive approximations to the desired behavior.
c. CORRECT The gradual reduction or withdrawal of prompts is called fading.
d. Incorrect Thinning is the process of gradually reducing the frequency of reinforcement (e.g., of switching from a continuous to an intermittent schedule of reinforcement).

The correct answer is: fading.

127
Q

Studies comparing the recall ability of participants who were or were not allowed to sleep following memorization of a list of words have provided support for which of the following theories of forgetting?
Select one:

A.
trace decay theory

B.
interference theory

C.
motivated forgetting theory

D.
retrieval failure theory

A

The research referred to in this question found that participants who slept remembered more than those who stayed awake.

a. Incorrect Trace decay theory predicts that forgetting is due to the passage of time. If this theory were correct, participants who did or did not sleep would have exhibited similar levels of recall.
b. CORRECT The results of this research suggest that forgetting is not due simply to the passage of time but, instead, to the events that follow learning. In other words, memory loss is due to the interference of other events.
c. Incorrect Freud proposed that motivated forgetting is due to the repression of unconscious desires.
d. Incorrect Retrieval failure theory predicts that a loss of memory is not due to a failure to store information in long-term memory but, instead, to an inability to retrieve that information.

The correct answer is: interference theory

128
Q

__________ used chimpanzees as his research subjects in studies designed to investigate the role of cognition in learning.
Select one:

A.
Tolman

B.
Ebbinghaus

C.
Thorndike

D.
Kohler

A

All of the individuals listed in the responses conducted studies on learning, but only one used chimpanzees as his research subjects.

a. Incorrect Tolman’s research with rats led to his identification of “latent learning.”
b. Incorrect Ebbinghaus acted as his own subject in research on forgetting.
c. Incorrect Thorndike’s research with cats in puzzle boxes led to the formulation of the “law of effect.”
d. CORRECT Kohler’s work with chimpanzees led to his description of insight learning.

The correct answer is: Kohler

129
Q

Just as you begin eating a hot fudge sundae, a friend describes what the sundae reminds him of in the most disgusting terms he can think of. After he does this several times, you cannot continue eating the sundae because it makes you feel sick. This situation is most similar to which of the following techniques?
Select one:

A.
operant shaping

B.
prompting and fading

C.
paradoxical intention

D.
covert sensitization

A

As the result of pairing an imagined aversive scene with a sundae, eating a sundae has became unpleasant and, as a result, “sundae-eating behavior” will be avoided. In this situation, the imagined aversive scene is the US, the sundae is the CS, and the avoidance response is the UR/CR.

a. Incorrect This situation is not similar to operant shaping, which uses reinforcement of successive approximations to a desired behavior to increase or establish that behavior.
b. Incorrect This situation is not similar to prompting and fading. Prompting is used to facilitate the acquisition of a new behavior. Verbal instructions, physical guidance, nonverbal cues are examples of prompts. Fading is the gradual removal of prompts.
c. Incorrect This situation is not similar to paradoxical intention, which involves imagining an exaggerated version of conditions surrounding a fear in order to eliminate the fear.
d. CORRECT This situation is most similar to covert sensitization, a type of aversive counterconditioning in which the aversive stimulus (US) is presented in imagination. The goal of covert sensitization is to replace the desirable or pleasant response to a CS with an undesirable or unpleasant response.

The correct answer is: covert sensitization

130
Q

The phenomenon known as “blocking” will occur in which the following circumstances?
Select one:

A.
The CS is presented prior to the US until a CR is established; then the CS and a new neutral stimulus are presented together several times prior to the US.

B.
The CS is presented prior to the US until a CR is established; then the CS is presented several times with a new neutral stimulus until the CR is extinguished.

C.
The CS is presented prior to the US until a CR is established; then a new neutral stimulus is presented prior to the CS.

D.
The CS is presented prior to the US until a CR is established; then a new neutral stimulus is presented after the CS.

A

Blocking occurs when a CS blocks the ability of a second neutral stimulus from becoming a CS.

a. CORRECT Blocking occurs when an association between the CS and the US has been established before the CS and a new neutral stimulus are presented together before the US. In this situation, the new neutral stimulus provides redundant information and, consequently, does not become associated with the US and does not produce a CR.
b. Incorrect See explanation above.
c. Incorrect See explanation above.
d. Incorrect See explanation above.

The correct answer is: The CS is presented prior to the US until a CR is established; then the CS and a new neutral stimulus are presented together several times prior to the US.

131
Q

Our eyes have near and distance focal points. As part of the normal aging process, we are most likely to experience which of the following?
Select one:

A.
our near focal point will move away from our eyes

B.
our near focal point will move closer to our eyes

C.
our near focal point will remain the same but our distance focal point will move away from our eyes

D.
our near focal point will remain the same but our distance focal point will move closer to our eyes

A

Humans experience several changes in sensory abilities as part of the normal aging process.

a. CORRECT Most adults begin to experience problems focusing on close objects by around age 40. This is referred to as presbyopia and occurs when the near focal point moves away from the eye.
b. Incorrect See explanation above.
c. Incorrect See explanation above.
d. Incorrect See explanation above.

The correct answer is: our near focal point will move away from our eyes

132
Q

Current theories of empathy development consider “emotional contagion” to be its earliest manifestation. This behavior emerges:
Select one:

A.
in the first weeks of life.

B.
between two and four months of age.

C.
at about six months of age.

D.
at about one year.

A

Emotional contagion refers to the tendency of babies to cry at the sound of another infant’s cries. This behavior is considered by experts to be the first sign of empathy.

a. CORRECT Emotional contagion is observed in infants as young as two days.
b. Incorrect See explanation above.
c. Incorrect See explanation above.
d. Incorrect See explanation above.

The correct answer is: in the first weeks of life.

133
Q

The intellectual disability associated with PKU is preventable with:
Select one:

A.
removal of environmental toxins.

B.
blood transfusions.

C.
a special diet.

D.
antibiotics.

A

PKU (phenylketonuria) is a recessive gene disorder.

a. Incorrect See explanation for response c.
b. Incorrect See explanation for response c.
c. CORRECT PKU involves an inability to metabolize the amino acid phenylalanine. Intellectual disability and other symptoms of PKU can be reduced or eliminated by a diet low in phenylalanine beginning at birth.
d. Incorrect See explanation for response c.

The correct answer is: a special diet.

134
Q

In terms of the parenting styles identified by Baumrind and her colleagues (1991), independent self-confident adolescents are most likely to have parents who are:
Select one:

A.
authoritative.

B.
authoritarian.

C.
permissive.

D.
traditional.

A

Baumrind and her colleagues distinguish between authoritarian, authoritative, permissive, and rejecting-neglecting parents. It’s very likely you’ll encounter a question on these styles on the exam, so you’ll want to be familiar with their characteristics and effects, which are described in the written study materials.

a. CORRECT Of the four parenting styles, the authoritative style is most likely to produce independent self-confident children and adolescents. Authoritative parents combine rational control with warmth, receptivity, and the encouragement of independence.
b. Incorrect Authoritarian parents impose absolute standards of control, stress obedience, and are willing to use force to obtain compliance. Their children tend to be insecure, anxious, and dependent and display a limited sense of responsibility.
c. Incorrect Permissive parents provide their children with few controls or demands and display moderate levels of warmth. Their children exhibit little self-reliance, exploratory behavior, or self-control.
d. Incorrect “Traditional” is not a parenting style identified by Baumrind.

The correct answer is: authoritative.

135
Q

Babbling ordinarily begins at about 4 to 5 months of age and initially includes:
Select one:

A.
phonemes from all languages.

B.
phonemes from the childs native language only.

C.
morphemes from all languages.

D.
morphemes that the child has heard most frequently.

A

Babbling involves the repetition of vowel and consonant sounds - e.g., bi-bi-bi and da-da-da.

a. CORRECT Babbling initially consists of phonemes from all languages; but, by about 9 months, it narrows to include the sounds of the child’s native language only.
b. Incorrect See explanation for response a.
c. Incorrect See explanation for response a.
d. Incorrect See explanation for response a.

The correct answer is: phonemes from all languages.

136
Q

During the second stage of Kohlberg’s preconventional level of moral development, children obey rules because:
Select one:

A.
they feel they have a personal duty to uphold rules and laws.

B.
everyone else is doing it.

C.
doing so helps them avoid punishment.

D.
doing so helps them satisfy their personal needs.

A

Kohlberg’s theory of moral development consists of three levels, with each level including two stages. For the exam, you want to be familiar with the characteristics of the six stages, which are described in the Lifespan Development chapter of the written study materials.

a. Incorrect This is characteristic of the second stage of Kohlberg’s conventional level.
b. Incorrect This doesn’t describe the second stage of the preconventional level.
c. Incorrect This is characteristic of the initial punishment and obedience stage of the preconventional level.
d. CORRECT Instrumental hedonism is the second stage of the preconventional level. Children in this stage consider the correct action to be the one that best satisfies their own personal needs. To illustrate this stage, Kohlberg described a 10-year old boy who, when asked what it means to be a good son, replied, “Be good to your father and he will be good to you.”

The correct answer is: doing so helps them satisfy their personal needs.

137
Q

A 16-month-old child who calls her pet cat “kitty” sees a dog for the first time and calls it “kitty.” In terms of Piaget’s theory of cognitive development, the child is displaying:
Select one:

A.
assimilation.

B.
accommodation.

C.
categorization.

D.
centration.

A

Piaget distinguished between two complementary processes that underlie cognitive development: assimilation and accommodation.

a. CORRECT Assimilation involves incorporating new knowledge into existing cognitive structures or schemes. The child in the question doesn’t know what a dog is and is incorporating it into her existing “kitty” scheme.
b. Incorrect Accommodation involves modifying an existing scheme or creating a new one.
c. Incorrect Piaget did not use the term “categorization” to describe the incorporation of new knowledge into an existing cognitive scheme.
d. Incorrect Centration is a limitation of the preoperational stage and refers to the tendency to focus on the most salient aspect of an object.

The correct answer is: assimilation.

138
Q

Research suggests children’s successful adjustment to the divorce of their parents is most related to:
Select one:

A.
the frequency of contact with the noncustodial parent.

B.
the quality of the parent-child relationships prior to the divorce.

C.
the childs ability to understand the cause(s) of the divorce.

D.
the degree of post-divorce conflict between the parents.

A

A number of child and parent factors have been linked to children’s adjustment to the divorce of their parents.

a. Incorrect See explanation for response d.
b. Incorrect See explanation for response d.
c. Incorrect See explanation for response d.
d. CORRECT Of the factors listed in the answers, the frequency and severity of post-divorce conflict between the parents has most consistently been linked to children’s adjustment. In fact, some experts argue that it is marital conflict - and not the divorce itself - that is responsible for children’s adjustment problems.

The correct answer is: the degree of post-divorce conflict between the parents.

139
Q

Thomas and Chess’s (1977) “goodness-of-fit” model predicts that maladjustment in children is due to:
Select one:

A.
a mismatch between the childs basic temperament and his/her parents childrearing practices.

B.
a mismatch between the childs attachment behaviors and his/her parents response to those behaviors.

C.
inconsistencies in the parenting styles of the child’s caregivers.

D.
inconsistencies in the childs sense of self and the feedback the child receives from caregivers about his/her behavior.

A

Knowing that Thomas and Chess studied temperament would have enabled you to identify the correct response to this question.

a. CORRECT Based on their research, Thomas and Chess concluded that it is not the child’s early temperament that predicts future adjustment but the match between the child’s temperament and his/her caregivers’ childrearing practices.
b. Incorrect See explanation for response a.
c. Incorrect See explanation for response a.
d. Incorrect See explanation for response a.

The correct answer is: a mismatch between the childs basic temperament and his/her parents childrearing practices.

140
Q

For most children, stranger anxiety begins at about _____ months of age.
Select one:

A.
2 to 3

B.
4 to 6

C.
8 to 10

D.
12 to 14

A

Stranger anxiety involves anxious, fearful responses in the presence of a stranger, especially when a caregiver is not present.

a. Incorrect See explanation for response c.
b. Incorrect See explanation for response c.
c. CORRECT Although the experts differ somewhat with regard to the exact age of onset of stranger anxiety, most agree that it begins during the second half of the first year.
d. Incorrect See explanation for response c.

The correct answer is: 8 to 10

141
Q

Alice, age 14, is certain that everyone is looking at her because she has a large blemish in the middle of her forehead. According to Elkind (1967), Alice’s belief is a manifestation of:
Select one:

A.
self-conscious emotions.

B.
the personal fable.

C.
social referencing.

D.
the imaginary audience.

A

Elkind (1967) proposed that adolescents experience a renewed egocentrisim that is manifested in several ways.

a. Incorrect Self-conscious emotions include shame, embarrassment, and pride and are not relevant to Elkind’s explanation for Alice’s behavior.
b. Incorrect The personal fable is one manifestation of adolescent egocentrism but does not describe Alice’s concern. It refers to the belief that one is special and unique and not subject to the same dangers as other people are.
c. Incorrect Social referencing refers to relying on the emotional response of another person (e.g., caregiver) to decide how to respond in an ambiguous situation.
d. CORRECT Elkind used the term “imaginary audience” to describe the adolescent’s belief that he/she is the focus of everyone’s attention.

The correct answer is: the imaginary audience.

142
Q

According to Bronfenbrenner’s (2004) ecological model, the __________ consists of interactions between elements of the microsystem (e.g., between the family and the school).
Select one:

A.
mesosystem

B.
exosystem

C.
macrosystem

D.
endosystem

A

Bronfenbrenner’s ecological model distinguishes between five layers (levels) that influence a child’s development: microsystem, mesosystem, exosystem, macrosystem, and chronosystem.

a. CORRECT The mesosystem is the second layer and consists of interactions between components of the microsystem - e.g., the parents’ involvement in the child’s school experiences and interactions between the child’s church and community.
b. Incorrect The exosystem consists of elements in the social setting that indirectly affect the child’s development (e.g., the parents’ jobs).
c. Incorrect The macrosystem is the outermost layer and includes the values, laws, customs, etc. of the child’s culture.
d. Incorrect The endosystem is not one of the levels identified by Bronfenbrenner.

The correct answer is: mesosystem

143
Q

In terms of age, Erikson’s ________ stage of psychosocial development corresponds to Freud’s phallic stage.
Select one:

A.
industry vs. inferiority

B.
autonomy vs. shame and doubt

C.
basic trust vs. mistrust

D.
initiative vs. guilt

A

Erikson’s psychosocial theory proposes that personality development occurs throughout the lifespan and involves eight stages and, for the exam, you want to be familiar with the names of the stages and the ages at which they occur.

a. Incorrect The industry vs. inferiority stage is characteristic of children aged 6 to 11 years and corresponds to Freud’s latency stage.
b. Incorrect The autonomy vs. shame and doubt stage occurs between the ages of 1 and 3 years and corresponds to Freud’s anal stage.
c. Incorrect The basic trust vs. mistrust stage occurs from birth to 1 year of age and corresponds to Freud’s oral stage.
d. CORRECT The initiative vs. guilt stage is characteristic of children aged 3 to 6 years and corresponds to Freud’s phallic stage.

The correct answer is: initiative vs. guilt

144
Q

A child with an insecure/avoidant pattern of attachment:
Select one:

A.
is very distressed when separated from his/her mother and continues to be highly anxious when she returns.

B.
is more anxious when his/her mother is present than absent and resists contact with her when she returns following separation.

C.
is very distressed when separated from his/her mother but pulls away from her when she returns.

D.
shows little distress when separated from his/her mother and turns away from her when she returns.

A

The four attachment patterns described by Ainsworth and her colleagues (secure, insecure/ambivalent, insecure/avoidant, and disorganized/disoriented) are described in the Lifespan Development chapter of the written study materials. You’ll want to become familiar with the characteristics associated with each pattern because you’re likely to encounter a question similar to this one on the exam.

a. Incorrect See explanation for response d.
b. Incorrect See explanation for response d.
c. Incorrect See explanation for response d.
d. CORRECT In Ainsworth’s “strange situation,” insecure/avoidant children are uninterested in exploration, show little distress when separated from their mothers, and avoid her when she returns.

The correct answer is: shows little distress when separated from his/her mother and turns away from her when she returns.

145
Q

Konrad Lorenz’s (1957) research on “imprinting” led to the search for ____________________ in human infants.
Select one:

A.
a critical period for attachment

B.
a biological explanation for language acquisition

C.
a sensitive period for social referencing

D.
a biological mechanism for recognition memory

A

Imprinting refers to the bond that develops during a critical period following birth in some non-human species.

a. CORRECT The ethologist Lorenz found that geese bonded to the first moving object they encounter during the first few days following birth. His research led to a search for a similar “critical period” for attachment in human infants.
b. Incorrect See explanation for response a.
c. Incorrect See explanation for response a.
d. Incorrect See explanation for response a.

The correct answer is: a critical period for attachment

146
Q

According to Gerald Patterson and his colleagues (1992), which of the following is a key contributor to aggressive behavior in children?
Select one:

A.
the childs intelligence

B.
the childs basic personality

C.
the strength of the parent-child attachment

D.
the parents childrearing behaviors

A

The social learning approach of Patterson and his colleagues focuses on the family factors that contribute to aggressiveness in children.

a. Incorrect See explanation for response d.
b. Incorrect See explanation for response d.
c. Incorrect See explanation for response d.
d. CORRECT Patterson et al.’s “coercive family interaction model” attributes aggression in children primarily to interactions between parents and their children, especially interactions involving the parents’ childrearing behaviors.

The correct answer is: the parents childrearing behaviors

147
Q

The __________ is the least developed area of the brain at birth.
Select one:

A.
brain stem

B.
cerebral cortex

C.
limbic system

D.
cerebellum

A

Not surprisingly, it is the higher centers of the brain that are least developed at birth.

a. Incorrect See explanation for response b.
b. CORRECT The cerebral cortex, which is responsible for higher-level cognitive skills, complex motor behaviors, language, and spatial skills, is almost completely undeveloped at birth.
c. Incorrect See explanation for response b.
d. Incorrect See explanation for response b.

The correct answer is: cerebral cortex

148
Q

Most babies say their first words at about _____ months of age.
Select one:

A.
7

B.
9

C.
12

D.
15

A

For the exam, you want to be familiar with the language milestones of early childhood that are described in the Lifespan Development chapter of the written study materials.

a. Incorrect See explanation for response c.
b. Incorrect See explanation for response c.
c. CORRECT Most infants say their first recognizable word at 12 months of age.
d. Incorrect See explanation for response c.

The correct answer is: 12

149
Q

Children begin to deliberately and regularly use rehearsal, elaboration, and organization as memory strategies by _____ years of age.
Select one:

A.
2 to 3

B.
5 to 7

C.
9 to 10

D.
13 to 15

A

The consistent and deliberate use of memory strategies does not occur until age 9 or 10.

a. Incorrect See explanation for response c.
b. Incorrect Preschoolers sometimes use memory strategies but do so in non-deliberate and often ineffective ways. In addition, children of this age can be taught to use memory strategies for a specific task but do not apply them to new tasks.
c. CORRECT The research has shown that the deliberate and consistent use of the memory strategies listed in the question begins at about 9 to 10 years of age.
d. Incorrect The use of these strategies continues to be “fine-tuned” during adolescence.

The correct answer is: 9 to 10

150
Q

_____________ involves intellectual disability and extreme obesity and is caused by a chromosomal deletion.
Select one:

A.
Down syndrome

B.
Turner syndrome

C.
Klinefelter syndrome

D.
Prader-Willi syndrome

A

Approximately 1 in 200 babies is born with a chromosomal abnormality.

a. Incorrect Down syndrome is attributable to the presence of an extra chromosome.
b. Incorrect Turner syndrome is caused by the absence of a chromosome. It occurs only in females and involves the absence of one X chromosome.
c. Incorrect Klinefelter syndrome occurs in males and is due to the presence of two or more X chromosomes and a single Y chromosome.
d. CORRECT Prader-Willi syndrome is caused by a chromosomal deletion, which occurs when part of a chromosome is missing.

The correct answer is: Prader-Willi syndrome

151
Q

The rooting reflex occurs when:
Select one:

A.
the newborns cheek is touched.

B.
the newborns foot is tickled.

C.
the newborn hears his/her caregivers voice.

D.
the newborn hears an unexpected loud noise.

A

Reflexes are unlearned responses to particular stimuli in the environment.

a. CORRECT The rooting reflex occurs when the baby turns his/her head in the direction of a touch or stroke on his/her cheek.
b. Incorrect This describes the Babinski reflex.
c. Incorrect This does not describe the rooting reflex.
d. Incorrect This describes the Moro (startle) reflex.

The correct answer is: the newborns cheek is touched.

152
Q

Auditory localization in children:
Select one:

A.
is fully developed by 2 or 3 months of age.

B.
does not begin to develop until the 8th or 9th month.

C.
has fully developed by about the end of the first year.

D.
shows a burst in development during the preschool years.

A

Auditory localization refers to the ability to orient toward the direction of a sound (i.e., by turning one’s head toward the sound).

a. Incorrect See explanation for response c.
b. Incorrect See explanation for response c.
c. CORRECT Infants exhibit some auditory localization during the first month of life but, between two and four months, there seems to be a loss of this ability. At four months, the ability returns, and by, one year, is completely (or almost completely) established.
d. Incorrect See explanation for response c.

The correct answer is: has fully developed by about the end of the first year.

153
Q

Longitudinal research by Shedler and Block (1990) found that drug use among adolescents is most linked with:
Select one:

A.
peer pressure to use drugs.

B.
lack of information about the effects of drugs.

C.
alienation, impulsivity, and subjective distress.

D.
early experimentation with drugs.

A

Although “just say no” and educational programs are probably the most popular types of anti-drug programs aimed at children and adolescents, the research suggests that their success is destined to be limited because they are based on the wrong assumptions regarding the causes of problem drug use.

a. Incorrect While peer pressure does underlie experimentation with drugs, it does not seem to be linked with problem drug use among adolescents.
b. Incorrect Lack of information has not been found to be a significant contributor to problem drug use.
c. CORRECT Based on their longitudinal study, J. Shedler and J. Block (Adolescent Drug Use and Psychological Health, American Psychologist, 45, 612-630, 1990) conclude that problem drug users exhibit a coherent syndrome, often as early as age 7, that is characterized by alienation, impulsivity, and subjective distress and that these symptoms clearly precede the onset of drug use.
d. Incorrect Experimentation is only a predictor of problem drug use in the presence of the syndrome described above.

The correct answer is: alienation, impulsivity, and subjective distress.

154
Q

Most infants take their first steps while holding onto furniture at about:
Select one:

A.
6 to 7 months of age.

B.
9 to 10 months of age.

C.
12 to 13 months of age.

D.
15 to 16 months of age.

A

Although children vary considerably in motor and other aspects of development, some generalizations can be made.

a. Incorrect See explanation for response b.
b. CORRECT At about 9 to 10 months, most babies are able to take a few steps while holding onto furniture. This is followed, at about 12 to 13 months, by the ability to walk without help or support.
c. Incorrect See explanation for response b.
d. Incorrect See explanation for response b.

The correct answer is: 9 to 10 months of age.

155
Q

At the beginning of the school year, a third grade teacher is told that ten of her students are “academic spurters” who can be expected to make substantial gains during the next few months. In fact, the ten children were randomly chosen. Most likely, these ten students:
Select one:

A.
will have made more gains than the other students in the class by the end of the school year.

B.
will be teased or ignored by the other students who are jealous of the special attention the students are receiving from the teacher.

C.
will exhibit a high degree of academic anxiety due to the pressure put on them by the teacher.

D.
will be indistinguishable from the other students at the end of the school year.

A

This question is asking about the Rosenthal effect, which is also known as the self-fulfilling prophecy effect and was originally described by Rosenthal and Jacobson (1968).

a. CORRECT Rosenthal and Jacobson found that telling a teacher that randomly selected children were expected to be academic spurters had a self-fulfilling prophecy effect. In other words, those children showed greater improvements at the end of the school year than children who were not identified as academic spurters.
b. Incorrect See explanation above.
c. Incorrect See explanation above.
d. Incorrect See explanation above.

The correct answer is: will have made more gains than the other students in the class by the end of the school year.

156
Q

A distinction is made between children who experience peer rejection versus peer neglect. Studies comparing these two groups of children suggest that changing schools:
Select one:

A.
has no effect on the peer status of rejected children but may have beneficial effects on the peer status of neglected children.

B.
has no effect on the peer status of neglected children but may have beneficial effects on the peer status of rejected children.

C.
is likely to have no effect on the peer status of either rejected or neglected children.

D.
is likely to have beneficial effects on the peer status of both rejected and neglected children.

A

The developmental trajectories of rejected and neglected children differ, with outcomes generally being worse for rejected children.

a. CORRECT Studies of rejected and neglected children have found that changing schools doesn’t help rejected children who continue to be rejected in the new environment. In contrast, when neglected children change schools, their peer status often changes. See J. D. Coie and J. B. Kupersmidt, A behavioral analysis of emerging social status in boys’ groups, Child Development, 1983, 54, 1400-1416.
b. Incorrect See explanation for response a.
c. Incorrect See explanation for response a.
d. Incorrect See explanation for response a.

The correct answer is: has no effect on the peer status of rejected children but may have beneficial effects on the peer status of neglected children.

157
Q

Which of the following is true about the sleep of older adults?
Select one:

A.
They have less REM sleep but more Stage 3 and Stage 4 sleep.

B.
They have a decreased need for sleep.

C.
They often experience an advanced sleep phase.

D.
They often experience a delayed sleep phase.

A

For the exam, you want to be familiar with the effects of age on sleep patterns that are described in the written study materials.

a. Incorrect Increasing age is associated with decreases in REM, Stage 3, and Stage 4 sleep.
b. Incorrect This is not true as evidenced by the fact that a lack of sleep at night is accompanied by increased sleepiness during the day.
c. CORRECT As we age, our circadian clock advances, causing an advanced sleep phase, which means that we get sleepier earlier in the evening and wake up earlier in the morning.
d. Incorrect See explanation for response c.

The correct answer is: They often experience an advanced sleep phase.

158
Q

Hypothyroidism is most associated with which of the following symptoms?
Select one:

A.
depressive symptoms, mental slowing, and memory impairment

B.
emotional lability, unrealistic fears and doubts, and impaired concentration

C.
agitated paranoia, confusion, and hallucinations and delusions

D.
unprovoked anger, episodic confusion, and lethargy or fatigue

A

Hypothyroidism is caused by underactivity of the thyroid gland. Like other endocrine disorders, hypothyroidism produces symptoms suggestive of a mental disorder.

a. CORRECT These are the classic symptoms of untreated hypothyroidism, which produces a slowing down of functioning and diminished capabilities.
b. Incorrect These symptoms are more suggestive of hyperthyroidism.
c. Incorrect These symptoms are characteristic of hyperparathyroidism.
d. Incorrect This sounds more like hypoglycemia.

The correct answer is: depressive symptoms, mental slowing, and memory impairment

159
Q

A patient is blindfolded and handed a familiar object in her left hand. She is unable to verbally identify the object. However, when the object is subsequently placed in her right hand, she can name it. This suggests that the woman has experienced damage to which of the following areas of her brain?
Select one:

A.
hippocampus

B.
cerebellum

C.
corpus callosum

D.
RAS

A

The question describes a typical task used to assess the effects of severing the corpus callosum.

a. Incorrect Although the hippocampus is involved with memory, damage to the hippocampus will not necessarily result in the behavior described in this question.
b. Incorrect The cerebellum governs coordinated movements.
c. CORRECT Research on “split-brain” patients has found that severing the corpus callosum limits the amount of information that can be transferred from one hemisphere to the other. When an object is placed in the left hand of a split-brain patient, the information goes directly to the right hemisphere but cannot then be transferred to the left hemisphere, which governs language.
d. Incorrect The RAS (reticular activating system) is involved in attention and arousal.

The correct answer is: corpus callosum

160
Q

The __________ is a key structure in the mediation of conditioned fear.
Select one:

A.
medulla

B.
thalamus

C.
amygdala

D.
RAS

A

Don’t let the term “conditioned fear” confuse you. To answer this question, you just need to know which of the four brain structures listed in the answers is associated with emotion.

a. Incorrect See explanation for response c.
b. Incorrect See explanation for response c.
c. CORRECT The research (including research investigating conditioned fear) has shown that fear is mediated by circuitry in the amygdala.
d. Incorrect See explanation for response c.

The correct answer is: amygdala

161
Q

The _____________ is the large band of nerve fibers that connects the left and right cerebral hemispheres and permits interhemispheric communication of sensory, motor, and higher-order information.
Select one:

A.
corpus callosum

B.
arcuate fasciculus

C.
cingulate gyrus

D.
globus pallidus

A

A question on the exam on the corpus calllosum may be relatively easy like this one or may be more difficult and require you to know how functioning is affected in a “split-brain patient” whose corpus callosum has been severed. These effects are described in the Physiological Psychology chapter of the written study materials.

a. CORRECT The corpus callosum is the largest band of fibers connecting the two hemispheres of the brain. Abnormalities in corpus callosum functioning have been linked to several disorders including dyslexia, ADHD, Tourette”s syndrome, and schizophrenia.
b. Incorrect The arcuate fasciculus connects Wernicke’s and Broca’s areas and carries signals related to the production and comprehension of language.
c. Incorrect The cingulate gyrus is part of the limbic system and is believed to be responsible for focusing attention on emotionally significant events, linking sensory input to emotions, mediating emotional reactions to pain, and regulating aggressive behavior.
d. Incorrect The globus pallidus is part of the basal ganglia and is involved in motor control.

The correct answer is: corpus callosum

162
Q

There is evidence that the _____________ plays a role in the etiology of seasonal affective disorder (SAD).
Select one:

A.
red nucleus

B.
suprachiasmatic nucleus

C.
tectum

D.
striatum

A

Seasonal affective disorder is a recurrent type of depression that occurs in the fall and winter months and has been linked to reduced exposure to daylight.

a. Incorrect The red nucleus is a midbrain structure and is involved in motor coordination.
b. CORRECT Knowing that the suprachiasmatic nucleus (SCN) mediates the sleep-wake cycle and other circadian rhythms may have helped you identify the correct answer to this question. The SCN’s role in SAD is believed to be related to its impact on melatonin levels, which are generally higher in individuals with this disorder and are affected by exposure to light.
c. Incorrect The tectum is part of the midbrain and is involved in auditory and visual processing.
d. Incorrect The striatum is part of the basal ganglia and is involved in movement and certain cognitive processes.

The correct answer is: suprachiasmatic nucleus

163
Q

Huntington’s disease is an:
Select one:

A.
autosomal dominant disorder.

B.
autosomal recessive disorder.

C.
X-linked dominant disorder.

D.
X-linked recessive disorder.

A

You’re likely to encounter a question on the exam the characteristics or causes of Huntington’s disease, so you’ll want to be familiar with the information on this disorder included in the Physiological Psychology chapter of the written study materials. Also be sure that you can distinguish between Huntington’s and Parkinson’s disease.

a. CORRECT The term “autosome” refers to a chromosome that is not an X or Y sex chromosome. Huntington’s disease is transmitted by a single dominant autosomal chromosome.
b. Incorrect See explanation above.
c. Incorrect See explanation above.
d. Incorrect See explanation above.

The correct answer is: autosomal dominant disorder.

164
Q

Disinhibition, apathy, and deficits in executive functioning are most associated with damage to the __________ lobes.
Select one:

A.
parietal

B.
frontal

C.
temporal

D.
occipital

A

You’re likely to encounter at least two or three questions on the exam that require you to be familiar with the functions of one of the four lobes of the cerebral cortex. These are described in the Physiological Psychology chaper of the written study materials.

a. Incorrect Damage to the parietal lobes affects somatosensory functioning.
b. CORRECT The prefrontal area of the frontal lobes is involved in a variety of complex behaviors including emotions, memory, self-awareness, and executive cognitive functions. Depending on its location, damage to this area can cause disinhibition (impulsivity, loss of tact), apathy (limited motivation and initiative), and/or deficits in higher-order cognitive skills.
c. Incorrect Temporal lobe damage may cause memory loss and auditory deficits.
d. Incorrect Damage to the occipital lobes produces disturbances in visual processing.

The correct answer is: frontal

165
Q

Which of the following is consistent with the predictions of the James-Lange theory?
Select one:

A.
I think, therefore I am.

B.
I’m trembling, so I must be scared.

C.
It’s noon so I must be hungry.

D.
That was a great birthday surprise, so I must be happy.

A

Knowing that the James-Lange theory is a peripheralist theory of emotion would have helped you identify the correct answer to this question.

a. Incorrect See explanation for response b.
b. CORRECT According to the James-Lange theory, the psychological experience of emotion follows bodily responses to external stimuli.
c. Incorrect See explanation for response b.
d. Incorrect See explanation for response b.

The correct answer is: I’m trembling, so I must be scared.

166
Q

The research has most consistently linked which of the following characteristics of the Type A personality pattern to an increased risk for coronary heart disease?
Select one:

A.
a sense of time urgency

B.
hostility and anger

C.
perfectionism

D.
competitiveness and ambitiousness

A

People with a Type A personality pattern are very competitive and achievement-oriented, have a sense of time urgency, and tend to be irritable, angry, and hostile.

a. Incorrect See explanation for response b.
b. CORRECT Most of the research on the Type A personality has found that, in general, it is not predictive of an increased risk for coronary heart disease. However, anger and hostility, which are characteristic of the Type A personality, have been linked to this disease and other stress-related illnesses.
c. Incorrect See explanation for response b.
d. Incorrect See explanation for response b.

The correct answer is: hostility and anger

167
Q

A person with Wernicke’s aphasia has impaired language comprehension and:
Select one:

A.
nonfluent and unintelligible speech.

B.
nonfluent but intelligible speech.

C.
fluent but unintelligible speech.

D.
fluent and intelligible speech.

A

For the exam, you want to be familiar with the characteristics of the five types of aphasia described in the written study materials – i.e., Wernicke’s, Broca’s, conduction, transcortical, and global.

a. Incorrect See explanation for response c.
b. Incorrect See explanation for response c.
c. CORRECT Wernicke’s aphasia is characterized by impairments in the ability to produce and comprehend language. People with this type of aphasia have trouble understanding written and spoken language and their speech is fluent but unintelligible because it contains multiple errors including sound and word substitutions.
d. Incorrect See explanation for response c.

The correct answer is: fluent but unintelligible speech.

168
Q

A(n) __________ seizure is characterized by a loss of consciousness without prominent motor symptoms.
Select one:

A.
atonic

B.
absence

C.
clonic

D.
simple partial

A

Each type of seizure listed in the answers is associated with a different etiology and combination of symptoms.

a. Incorrect Atonic seizures involve a sudden loss of muscle tone that may cause the individual to fall down. The individual usually remains conscious during the seizure.
b. CORRECT The question accurately describes an absence (petit mal) seizure.
c. Incorrect Clonic seizures are characterized by jerky movements.
d. Incorrect A simple partial seizure begins on one side of the brain and, at least initially, affects only one side of the body. It causes changes in movement and sensation but does not produce a loss of consciousness.

The correct answer is: absence

169
Q

Agraphia, acalculia, finger agnosia, and right-left confusion are symptoms of which of the following?
Select one:

A.
Creutzfeldt-Jakob disease

B.
Addisons disease

C.
Gerstmanns syndrome

D.
Geschwind syndrome

A

The four symptoms listed in this question are caused by damage to the left (dominant) angular gyrus, which is located at the point of convergence of the parietal, occipital, and temporal lobes.

a. Incorrect Creutzfeldt-Jakob disease is a rare degenerative brain disease that usually leads to dementia and is characterized by memory loss, behavioral changes, loss of coordination, and visual disturbances.
b. Incorrect Addison’s disease is due to a lower-than-normal production of cortisol by the adrenal glands. Its symptoms include weight loss, orthostatic hypotension, weakness, fatigue, and hyperpigmentation.
c. CORRECT These are the characteristic symptoms of Gerstmann’s syndrome.
d. Incorrect Gerschwind syndrome is a personality syndrome associated with temporal lobe epilepsy and is characterized by circumstantiality, hypergraphia, alterations in sexuality, emotional volatility, and, in some cases, hyperreligiosity.

The correct answer is: Gerstmanns syndrome

170
Q

A person with a temporal lobe lesion will most likely exhibit which of the following?
Select one:

A.
memory impairment, depression or anxiety, and hallucinations

B.
poor judgment, impulsivity, and inappropriate jocularity

C.
homonymous hemianopia, prosopagnosia, and irritability and suspiciousness

D.
dysgraphia, depression and apathy, and denial of his/her problems

A

The symptoms listed in each answer to this question are associated with lesions in a different lobe of the cerebral cortex.

a. CORRECT The temporal lobe is important for auditory perception and long-term memory, and a lesion in this lobe affects memory, may produce hallucinations (especially auditory hallucinations), and often affects mood. Note that knowing that the temporal lobe plays a role in memory would have helped you identify this as the correct answer even if you’re unfamiliar with its other functions.
b. Incorrect These symptoms are associated with lesions in certain areas of the frontal lobe.
c. Incorrect These symptoms are more likely to be caused by an occipital lobe lesion.
d. Incorrect A lesion in the parietal lobe may produce these symptoms.

The correct answer is: memory impairment, depression or anxiety, and hallucinations

171
Q

As the result of a brain tumor, Alma A. is no longer able to recognize her family members or friends by looking at their faces. Alma’s condition is known as:
Select one:

A.
paresthesia.

B.
anosmia.

C.
prosopagnosia.

D.
anosognosia.

A

Alma is unable to recognize familiar faces, a condition that is caused by damage to certain areas of the occipital and temporal lobes.

a. Incorrect Paresthesia is an altered sensation in the skin that causes numbness or tingling.
b. Incorrect Anosmia is the absence of the sense of smell.
c. CORRECT Prosopagnosia is a type of visual agnosia that involves an inability to recognize familiar faces, often including one’s own face.
d. Incorrect Anosognosia is the inability to recognize (or the denial of) one’s own neurological symptoms.

The correct answer is: prosopagnosia.

172
Q

Intense hunger, headaches, anxiety, dizziness, weakness, heart palpitations, and confusion are symptoms of which of the following endocrine disorders?
Select one:

A.
hyperthyroidism

B.
hyperglycemia

C.
hypothyroidism

D.
hypoglycemia

A

The endocrine disorders produce a combination of physical and neuropsychiatric symptoms.

a. Incorrect Hyperthyroidism is characterized by heat intolerance, tachycardia, hyperactive reflexes, distractibility, and impaired problem-solving.
b. Incorrect Symptoms of hyperglycemia (high blood sugar) include polydipsia, polyuria, polyphagia, fatigue, weight loss, and recurrent infections.
c. Incorrect Hypothyroidism is characterized by cold intolerance, bradycardia, fatigue, mental slowing, and decreased libido.
d. CORRECT The symptoms described in this question are characteristic of hypoglycemia (low blood sugar), which has several causes including excessive production of insulin by the pancreas.

The correct answer is: hypoglycemia

173
Q

Early memory loss associated with Alzheimer’s dementia is believed to be caused by deterioration of neurons in the hippocampus that secrete:
Select one:

A.
GABA.

B.
serotonin.

C.
glycine.

D.
acetylcholine.

A

For the exam, you’ll want to be familiar with the functions of the major neurotransmitters that are described in the Physiological Psychology chapter of the written study materials.

a. Incorrect GABA is an inhibitory neurotransmitter. It is believed to play a role in anxiety, sleep, and eating disorders and is affected by CNS depressants.
b. Incorrect Serotonin has been linked to a variety of functions including memory. However, abnormalities in serotonin levels have not been as consistently linked to the early memory loss of Alzheimer’s dementia as have been abnormalities in acetylcholine. Instead, serotonin is believed to contribute to other symptoms of this disorder including sensory disturbances, mood swings, and sleep disruption.
c. Incorrect Glycine plays a role in spinal reflexes and motor behavior.
d. CORRECT Acetylcholine (ACh) mediates motor functions as well as memory processes, and decreased levels of ACh in the hippocampus have been linked to the memory loss associated Alzheimer’s dementia.

The correct answer is: acetylcholine.

174
Q

Damage to the hippocampus is most likely to interfere with the ability to:
Select one:

A.
recall information stored in remote long-term memory.

B.
manipulate information while it is in short-term (working) memory.

C.
transfer information from short- to long-term memory.

D.
retrieve implicit memories, even when given adequate cues.

A

For the exam, you want to have the hippocampus linked to memory consolidation.

a. Incorrect Although the hippocampus contributes to memory consolidation, it does not appear to be involved in the storage of long-term memories.
b. Incorrect The hippocampus has not been found to contribute to the ability to manipulate information in working memory.
c. CORRECT The research has shown that the hippocampus is involved in explicit (conscious) memory and, more specifically, in transferring explicit memories from short- to long-term memory.
d. Incorrect The hippocampus appears to be more involved in explicit than in implicit memory.

The correct answer is: transfer information from short- to long-term memory.

175
Q

Eating aged cheese, soy products, processed meats, fermented foods, or other foods containing tyramine while taking an MAOI is contraindicated because doing so can cause which of the following?
Select one:

A.
rebound effect

B.
agranulocytosis

C.
tardive dyskinesia

D.
hypertensive crisis

A

Combining an MAOI (a type of antidepressant) with certain foods or other drugs can produce potentially fatal side effects.

a. Incorrect See explanation for response d.
b. Incorrect See explanation for response d.
c. Incorrect See explanation for response d.
d. CORRECT Taking an MAOI in combination with foods containing tyramine can cause a hypertensive crisis which involves elevated blood pressure, severe headache, nausea and vomiting, palpitations, blurred vision, stiff neck, sweating, and confusion.

The correct answer is: hypertensive crisis

176
Q

Agranulocytosis, a potential side effect of the atypical antipsychotic clozapine (Clozaril), is a ______ disorder.
Select one:

A.
seizure

B.
hormonal

C.
gastrointestinal

D.
blood

A

One of the disadvantages of some of the atypical antipsychotic drugs is that they may cause agranulocytosis.

a. Incorrect See explanation for response d.
b. Incorrect See explanation for response d.
c. Incorrect See explanation for response d.
d. CORRECT Agranulocytosis is an abnormally low level of white blood cells. It is a rare but dangerous side effect of clozapine.

The correct answer is: blood

177
Q
Propranolol hydrochloride (Inderal) is most likely to be prescribed as a treatment for which of the following?
Select one:

A.
hypotension

B.
essential tremor

C.
bronchial asthma

D.
diabetes

A

For the exam, you want to know that propranolol hydrochloride is a beta-blocker and is used to treat cardiovascular disorders, tremors, migraine headaches, and the physical symptoms of anxiety. You also want to be familiar with its side effects, which are described in the written study materials.

a. Incorrect Propranolol HCl is used to treat hypertension and is contraindicated for hypotension.
b. CORRECT Essential tremor (tremor associated with purposeful movement) is one of the conditions that propranolol HCl is used to treat.
c. Incorrect Propranolol HCl is contraindicated for bronchial asthma.
d. Incorrect Propranolol HCl is not used as a treatment for diabetes.

The correct answer is: essential tremor

178
Q

The research suggests that RNA (ribonucleic acid) plays an important role in which of the following?
Select one:

A.
the sleep-wake cycle

B.
learning and memory

C.
creativity

D.
motivation

A

RNA is involved in protein synthesis and other cell activities.

a. Incorrect See explanation for response b.
b. CORRECT The results of research indicate that the coding of memory may involve changes in protein molecules and, consequently, that RNA acts as a chemical mediator for learning and memory. RNA has not been linked to the sleep-wake cycle, creativity, or motivation.
c. Incorrect See explanation for response b.
d. Incorrect See explanation for response b.

The correct answer is: learning and memory

179
Q

Jennifer J. always experiences certain colors in response to certain sounds (e.g., the meow of a cat always elicits a reddish-pink color while the chirp of a bird elicits light blue). Jennifer’s experience is referred to as:
Select one:

A.
synesthesia.

B.
somesthesis.

C.
anosognosia.

D.
alexithymia

A

For Jennifer, stimulation of one sensory modality triggers sensation in another sensory modality.

a. CORRECT Synesthesia occurs when two or more senses are automatically and involuntarily joined so that stimulation of one sense automatically elicits another sense (e.g., a certain sound elicits a specific color).
b. Incorrect Somesthesis refers to the sensory perception associated with the skin, muscles, joints, and viscera and includes touch-pressure, warmth-coolness, and proprioception.
c. Incorrect Anosognosia is a lack of awareness of one’s own disability or illness.
d. Incorrect Alexithymia is an inability to describe one’s own feelings.

The correct answer is: synesthesia.

180
Q

Generally, the __________ branch of the autonomic nervous system activates bodily functions while the __________ branch returns the body to a resting state.
Select one:

A.
peripheral; central

B.
central; peripheral

C.
sympathetic; parasympathetic

D.
parasympathetic; sympathetic

A

The autonomic nervous system is divided into two branches: the parasympathetic and sympathetic.

a. Incorrect See explanation for response c.
b. Incorrect See explanation for response c.
c. CORRECT The sympathetic nervous system mediates arousal and expenditure of energy and prepares the body for action, while the parasympathetic nervous system allows the body to increase its store of energy and is active during states of relaxation.
d. Incorrect See explanation for response c.

The correct answer is: sympathetic; parasympathetic

181
Q

The potential adverse side effects of the benzodiazepines include:
Select one:

A.
insomnia, appetite loss, and restlessness.

B.
confusion, memory loss, and psychomotor impairment.

C.
muscle rigidity, catalepsy, and clammy skin.

D.
bradycardia, shortness of breath, and depression.

A

Knowing that the benzodiazepines are anxiolytics that are used primarily as treatments for anxiety and sleep disturbances would have helped you identify the correct answer to this question.

a. Incorrect These are common side effects of the psychostimulant drugs.
b. CORRECT Common side effects of the benzodiazepines include drowsiness, impaired psychomotor ability, disorientation and confusion, anterograde amnesia, and depression.
c. Incorrect These are signs of narcotic-analgesic overdose.
d. Incorrect These are side effects of the beta-blocker propranolol.

The correct answer is: confusion, memory loss, and psychomotor impairment.

182
Q

A split-brain patient will be able to do which of the following?
Select one:

A.
say the word “spoon” after it is projected to his left visual field

B.
repeat a series of letters that have been whispered in his left ear

C.
verbally identify an odor that has been presented to his right nostril

D.
identify a spoon with his left hand after an image of a spoon is projected to his left visual field

A

When answering questions about split-brain patients, keep in mind that, in most people, language is controlled by the left hemisphere and that most functions are controlled contralaterally (an exception is olfaction).

a. Incorrect Because information projected to the left visual field would go directly to the right hemisphere, a split-brain patient would not be able to say what he/she has just seen.
b. Incorrect Information entering the left ear is sent to the right hemisphere and, consequently, the split-brain patient cannot repeat that information.
c. Incorrect As noted above, olfactory signals do not cross to the other side of the brain. Consequently, stimulation entering the right nostril will be sent to the right hemisphere.
d. CORRECT An image presented to the left visual field will be processed in the right hemisphere, which controls the left hand. Therefore, a split-brain patient will be able to perform this task.

The correct answer is: identify a spoon with his left hand after an image of a spoon is projected to his left visual field

183
Q

When screening a patient for brain damage, a psychologist administers the Wisconsin Card Sort and the Halstead-Reitan Category Test. Apparently, the psychologist believes that the patient has:
Select one:

A.
diffuse brain damage.

B.
frontal lobe damage.

C.
temporal lobe damage.

D.
basal ganglia damage.

A

Being familiar with the purpose of one of the two tests listed in the question would have helped you identify the correct answer to this question.

a. Incorrect See explanation for response b.
b. CORRECT The frontal lobes are involved in a number of functions including initiative, planning, and completing tasks in sequence. Performance on the measures listed in the question (Wisconsin Card Sort, Category Test) depends on intact frontal lobe functioning.
c. Incorrect See explanation for response b.
d. Incorrect See explanation for response b.

The correct answer is: frontal lobe damage.

184
Q

Holland’s theory about the importance of matching a person’s personality with his/her work environment predicts that a person with a realistic orientation will be most satisfied with job tasks that:
Select one:

A.
are ambiguous and require some degree of creativity.

B.
require manual or technical skills and concrete problem-solving.

C.
require systematic manipulation and organization of data.

D.
require introspection and intellectual problem-solving.

A

Holland’s six personality types are realistic, investigative, artistic, social, enterprising, and conventional (“RIASEC”).

a. Incorrect This type of job is appropriate for an artistic individual.
b. CORRECT Realistic jobs require mechanical, manual, and technical skills.
c. Incorrect This type of job is suited to a person who is conventional.
d. Incorrect This describes the investigative type.

The correct answer is: require manual or technical skills and concrete problem-solving.

185
Q

The Peabody Picture Vocabulary Test (PPVT) was designed as a measure of receptive vocabulary for:
Select one:

A.
adolescents and adults with a visual impairment.

B.
children and adolescents with a hearing impairment.

C.
children, adolescents, and adults with autism.

D.
children, adolescents, and adults with a motor or speech impairment.

A

For the licensing exam, you’ll want to be familiar with the tests designed for individuals with physical disabilities that are described in the Psychological Assessment chapter of the study materials.

a. Incorrect The Haptic Intelligence Scale was designed for adolescents and adults with visual impairments.
b. Incorrect The Hiskey-Nebraska Test is a measure of intelligence for children and adolescents with hearing impairments.
c. Incorrect The PPVT was not originally designed for examinees with autism.
d. CORRECT The PPVT requires the examinee to point to (or otherwise indicate) the picture that best corresponds to the word verbally provided by the examiner and was originally developed for examinees with a motor or speech impairment.

The correct answer is: children, adolescents, and adults with a motor or speech impairment.

186
Q

The developer of a new occupational interest test conducts a study to determine which test items accurately distinguish between people who are employed in different occupations. This test developer is using which of the following methods of scale construction?
Select one:

A.
empirical criterion keying

B.
homogeneous keying

C.
factor analysis

D.
multiple regression analysis

A

The test developer described in this question is trying to determine which items distinguish between specific criterion groups (i.e., people in different occupations).

a. CORRECT Empirical criterion keying involves choosing items for different scales on the basis of how well the items distinguish between different criterion groups. This method was used in the development of the Occupational scales of the Strong Interest Inventory.
b. Incorrect When using homogeneous keying, items are selected on the basis of the degree to which they measure the same domain.
c. Incorrect When factor analysis is used for scale construction, items that correlate highly with the same factor are included in the same scale.
d. Incorrect Multiple regression analysis is a multivariate technique that is useful when performance on two or more predictors will be used to predict or estimate an individual’s score on a criterion. It is not a method of scale construction.

The correct answer is: empirical criterion keying

187
Q

To assess the general intelligence of a six-year-old child with a severe hearing impairment, you would use which of the following tests?
Select one:

A.
Halstead-Reitan

B.
KABC-II

C.
Haptic Intelligence Test

D.
Hiskey-Nebraska

A

Of the tests listed in the answers, only one is useful for evaluating the intelligence of a child with impaired hearing.

a. Incorrect The Halstead-Reitan is used to identify individuals with neurological impairment.
b. Incorrect The KABC-II is a measure of cognitive ability for individuals aged 3 to 18. Administration of the KABC-II involves verbal instructions, so it would not be useful for children with hearing impairments.
c. Incorrect The Haptic Intelligence Test is a measure of intelligence for adolescents and adults with impaired vision.
d. CORRECT A number of tests have been developed as alternatives to the Stanford-Binet and Wechsler tests for assessing the general intellectual ability of individuals with disabilities. The Hiskey-Nebraska test was developed for use with children aged 3 to 17 who have hearing impairments.

The correct answer is: Hiskey-Nebraska

188
Q

On the Rorschach Inkblot Test, confabulation is most suggestive of which of the following?
Select one:

A.
giftedness

B.
antisocial personality disorder

C.
an attempt to fake bad

D.
organic brain dysfunction

A

Confabulation on the Rorschach refers to the tendency to overgeneralize from a part of an inkblot to the entire inkblot.

a. Incorrect See explanation for response d.
b. Incorrect See explanation for response d.
c. Incorrect See explanation for response d.
d. CORRECT Confabulation is associated with cognitive impairment, especially impairment related to organic brain dysfunction.

The correct answer is: organic brain dysfunction

189
Q

The concordance rate for IQ for biological (non-twin) siblings who are reared together is:
Select one:

A.
.85.

B.
.67.

C.
.45.

D.
.22.

A

For the licensing exam, you’ll want to be familiar with the IQ concordance rates listed in the Psychological Assessment chapter of the written study materials.

a. Incorrect Bouchard and Gue (1981) report a concordance rate of .85 for IQ for identical twins reared together.
b. Incorrect The concordance rate for identical twins reared apart is .67.
c. CORRECT Bouchard and Gue report a concordance rate of .45 for biological (non-twin) siblings reared together.
d. Incorrect The concordance rate for biological parent and child living apart is .22.

The correct answer is: .45.

190
Q

In the case of Larry P. v. Riles, the court ruled that IQ tests:
Select one:

A.
are culturally fair.

B.
are racially and culturally biased.

C.
are valid predictors of job performance for members of minority groups when used in conjunction with other measures.

D.
are valid measures of intelligence for members of minority groups when administered by an examiner of the same race/ethnicity.

A

In the Larry P. case, a suit was brought against the California State Department of Education and the San Francisco School Board on the ground that the use of standard IQ tests as a placement technique resulted in an over-representation of African American children in special education classes.

a. Incorrect See explanation for response b.
b. CORRECT Judge Peckham”s ruling in the case of Larry P. reflected his conclusion that IQ tests are racially and culturally biased and, therefore, should not be used as placement tools for minority children.
c. Incorrect See explanation for response b.
d. Incorrect See explanation for response b.

The correct answer is: are racially and culturally biased.

191
Q

The Bender-Gestalt II was designed to be a measure of:
Select one:

A.
fine motor skills.

B.
visual-motor integration.

C.
cognitive flexibility.

D.
suicide risk.

A

The Bender-Gestalt II is a brief assessment of visual-motor integration.

a. Incorrect See explanation above.
b. CORRECT The Bender-Gestalt II is a drawing test that requires the examinee to copy several geometric figures. It is used to assess visual-motor integration and to assist in the assessment of perceptual disorders, organic brain dysfunction, and psychopathology.
c. Incorrect See explanation above.
d. Incorrect See explanation above.

The correct answer is: visual-motor integration.

192
Q

A psychologist administers the MMPI-2 to a 36-year old man who exhibits a number of symptoms that are suggestive of neurosis. The man receives a very low K Scale score, and, on the basis of this score, the psychologist can tentatively conclude that the man:
Select one:

A.
tried to present himself in a favorable light.

B.
is very self-critical.

C.
has a personality disorder.

D.
may have an organic brain disorder.

A

The K (Correction) Scale is one of the MMPI’s validity scales. For the exam, you want to be familiar with all of the MMPI validity that are described in the Psychological Assessment chapter of the written study materials.

a. Incorrect An elevated K score might indicate an attempt to “fake good.”
b. CORRECT A very low score on the K scale indicates excessive frankness or self-criticism or an attempt to “fake bad.”
c. Incorrect The MMPI-2’s validity scales were not designed to provide information about personality disorders.
d. Incorrect The MMPI-2’s validity scales were not designed to detect organicity.

The correct answer is: is very self-critical.

193
Q

Elmo E. is in the process of buying a new car and is having trouble deciding between two cars that are similar in terms of both positive and negative characteristics. In other words, Elmo is facing a(n) ________ conflict.
Select one:

A.
approach-approach

B.
avoidance-avoidance

C.
single approach-avoidance

D.
double approach-avoidance

A

For the exam, you want to be familiar with the four types of conflicts listed in the answers to these questions. Additional information about these conflicts is presented in the Social Psychology chapter of the written study materials.

a. Incorrect An approach-approach conflict occurs when a person has to choose between two equally positive events or objects. In this case, the two cars have both positive and negative characteristics, so this is not the best answer.
b. Incorrect An avoidance-avoidance conflict occurs when a person has to choose between two equally undesirable events or objects. Again, because the cars have both positive and negative qualities, this is not the best answer.
c. Incorrect A single approach-avoidance conflict occurs when a single object or event has both positive and negative qualities.
d. CORRECT As its name implies, a double approach-avoidance conflict occurs when a person has to choose between two objects or events that have both positive and negative qualities, which is the case in the situation described in this question.

The correct answer is: double approach-avoidance

194
Q

An employee expects greater consistency in the behavior of his boss than is warranted. The employee’s expectation is best accounted for by which of the following?
Select one:

A.
overjustification hypothesis

B.
self-serving bias

C.
fundamental attribution bias

D.
self-verification theory

A

This question is vague, and you may have had to rely on the process of elimination to identify the correct answer.

a. Incorrect The overjustification hypothesis predicts that internal motivation to perform a specific action is weakened when one is given external reinforcement for performing the action.
b. Incorrect The self-serving bias applies to the attributions one makes about one’s own behavior and is not relevant to the situation described in this question.
c. CORRECT The fundamental attribution bias is the tendency to attribute another person’s behavior to dispositional factors (e.g., traits) and, therefore, to expect a great deal of consistency in behavior.
d. Incorrect Self-verification theory predicts that people prefer receiving information about themselves from others that is consistent with their self-evaluations.

The correct answer is: fundamental attribution bias

195
Q

Which of the following best illustrates deindividuation?
Select one:

A.
A usually quiet, reserved person acts uncharacteristically violent in a crowd because he is able to do so anonymously.

B.
A victim of a crime is more likely to receive assistance when the crime occurs in a rural area rather than in a city.

C.
In a group decision-making situation, there is a tendency for critical-thinking to be suspended when group cohesiveness is very strong.

D.
A group member reduces his/her effort when he/she thinks other members are not exerting maximum effort.

A

Deindividuation occurs in conditions that foster a loss of personal identity.

a. CORRECT This is the kind of behavior that was originally linked with deindividuation - i.e., the research found that people who are normally non-aggressive may act in aggressive ways in crowds as the result of their ability to act anonymously.
b. Incorrect This describes bystander apathy.
c. Incorrect This sounds like groupthink.
d. Incorrect This is referred to as the sucker effect.

The correct answer is: A usually quiet, reserved person acts uncharacteristically violent in a crowd because he is able to do so anonymously.

196
Q

With regard to attitude change, inoculation is useful for:
Select one:

A.
increasing a persons resistance to persuasion.

B.
reducing a communicator’s anxiety prior to delivering a message to a hostile audience.

C.
distracting a person from a communicator’s message.

D.
ensuring that attitude change represents internalization rather than mere compliance.

A

The concept of inoculation (McGuire, 1969) was derived from the medical model and is based on the assumption that a person will be better able to resist a persuasive communication when he/she has been “inoculated” against it. Inoculation involves providing weak arguments against a position and counterarguments refuting those arguments.

a. CORRECT Inoculation has been shown to be an effective method for increasing an individual’s resistance to persuasion. McGuire found, for example, that it is more effective than providing the individual only with arguments that support his/her initial position.
b. Incorrect This is not the goal of inoculation.
c. Incorrect Distraction is associated with increased persuasiveness when a listener is initially opposed to a message and decreased persuasiveness when a listener is initially in favor of a message. Inoculation does not involve distraction.
d. Incorrect This is not a goal of inoculation.

The correct answer is: increasing a persons resistance to persuasion.

197
Q

Superordinate goals have been found useful for:
Select one:

A.
clarifying roles.

B.
maximizing satisfaction and motivation.

C.
reducing intergroup conflict.

D.
increasing feelings of self-efficacy.

A

For the exam, you want to be familiar with the definition of superordinate goals, what impact they have, and the classic research on these goals. These topics are all described in the Social Psychology chapter of the written study materials.

a. Incorrect See explanation for response c.
b. Incorrect See explanation for response c.
c. CORRECT Superordinate goals can be achieved only through intergroup cooperation. In their Robber’s Cave study, Sherif and his colleagues (1961) found that the introduction of superordinate goals was the most effective way for reducing intergroup hostility.
d. Incorrect See explanation for response c.

The correct answer is: reducing intergroup conflict.

198
Q

According to Bem’s (1972) self-perception theory:
Select one:

A.
people experience discomfort (dissonance) when their behaviors are not consistent with their attitudes, beliefs, or values.

B.
people enhance their own sense of self-worth by associating or identifying with others who are successful.

C.
people tend to attribute their successes to dispositional factors and failures to situational factors.

D.
people infer their own attitudes and emotions by observing their own behaviors and the circumstances in which those behaviors occur.

A

Self-perception theory predicts that people judge or determine their own internal states in the same way that they determine the states of other people - i.e., by considering external cues.

a. Incorrect This is predicted by cognitive dissonance theory but not by self-perception theory.
b. Incorrect This is not a prediction of self-perception theory.
c. Incorrect This answer describes the self-serving bias.
d. CORRECT According to self-perception theory, we’re most likely to rely on external information for determining or judging our internal states when internal cues are ambiguous, weak, or difficult to interpret.

The correct answer is: people infer their own attitudes and emotions by observing their own behaviors and the circumstances in which those behaviors occur.

199
Q

The tendency to seek out and interpret information in ways that verify our existing beliefs is referred to as the:
Select one:

A.
hindsight bias.

B.
primacy effect.

C.
focusing effect.

D.
confirmation bias.

A

Even if you’re unfamiliar with the cognitive bias being asked about by this question, you may have been able to identify the correct answer (confirmation bias) because it “sounds like” what is it.

a. Incorrect The hindsight bias refers to the tendency to believe that, after an outcome occurs, you could have predicted the outcome beforehand.
b. Incorrect The primacy effect refers to the tendency to remember stimuli that occurred first better than stimuli that occurred last.
c. Incorrect The focusing effect is the tendency to place too much importance (focus) on one aspect of an event or situation when predicting a future outcome.
d. CORRECT The confirmation bias is a cognitive bias which, as its name implies, involves focusing on and seeking out information that confirms our preconceptions.

The correct answer is: confirmation bias.

200
Q

The theory of planned behavior (Ajzen, 1991) predicts that:
Select one:

A.
attitudes are the result of behaviors.

B.
attitudes are the result of subjective norms, behavioral intentions, and behaviors.

C.
behaviors are the result of attitudes.

D.
behaviors are the result of attitudes, subjective norms, and behavioral intentions.

A

Ajzen’s (1991) theory of planned behavior (also known as the theory of reasoned action) predicts that behaviors are the result of several factors.

a. Incorrect See explanation for response d.
b. Incorrect See explanation for response d.
c. Incorrect This answer is only partially correct and, therefore, not the best answer of those given.
d. CORRECT The theory of planned behavior attributes behaviors to three factors: attitudes toward the behavior; beliefs about people’s opinions of the behavior and motivation to comply with people’s expectations (subjective norms); and the intention to perform the behavior (behavioral intention).

The correct answer is: behaviors are the result of attitudes, subjective norms, and behavioral intentions.

201
Q

The three components of attitudes are:
Select one:

A.
affect, cognition, and behavior.

B.
aptitude, affect, and cognition.

C.
evaluation, experience, and application.

D.
knowledge, evaluation, and action.

A

Attitudes are relatively stable evaluative responses to an entity or situation and are often described as consisting of three components.

a. CORRECT Most researchers interested in attitudes describe them as consisting of three components: affective (evaluative), cognitive, and behavioral (conative).
b. Incorrect See explanation for response a.
c. Incorrect See explanation for response a.
d. Incorrect See explanation for response a.

The correct answer is: affect, cognition, and behavior.

202
Q

Research suggests that people who frequently view TV shows that depict violent acts:
Select one:

A.
fear their environment more than people who report being less frequent viewers.

B.
fear their environment somewhat less than people who report being less frequent viewers.

C.
fear their environment much less than people who report being less frequent viewers.

D.
are about equal in terms of fear of their environment to people who report being less frequent viewers.

A

Studies on the effects of media violence on attitudes and behavior have confirmed the predictions of social learning theory.

a. CORRECT The results of the studies are not entirely consistent but, overall, they indicate that frequent viewing of media violence is associated with an increased fear of the environment.
b. Incorrect See explanation for response a.
c. Incorrect See explanation for response a.
d. Incorrect See explanation for response a.

The correct answer is: fear their environment more than people who report being less frequent viewers.

203
Q

A researcher would use the randomized block ANOVA to analyze the data she has collected in order to:
Select one:

A.
analyze the main and interaction effects of an extraneous variable.

B.
control measurement (random) error.

C.
evaluate the effects of an independent variable on multiple dependent variables.

D.
evaluate the effects of one or more independent variables on a nominal dependent variable.

A

As its name suggests, the randomized block ANOVA is the appropriate inferential statistical test when blocking has been used to control an extraneous variable.

a. CORRECT When using the randomized block ANOVA, the extraneous (“blocking”) variable is treated like an independent variable so that its main and interaction effects on the dependent variable can be evaluated.
b. Incorrect A randomized block ANOVA would not be useful for this purpose.
c. Incorrect A MANOVA would be useful for this purpose.
d. Incorrect The chi-square test would be useful for this purpose.

The correct answer is: analyze the main and interaction effects of an extraneous variable.

204
Q

Heteroscedasticity in a scattergram suggests that:
Select one:

A.
the relationship between the predictor and criterion cannot be described by a straight line.

B.
there is a restriction of range of scores on the predictor and/or the criterion.

C.
the variability (range) of scores on the criterion varies for different scores on the predictor.

D.
there is a statistically significant correlation between the predictor and criterion.

A

Heteroscedasticity means unequal variability.

a. Incorrect This is not a conclusion that can be drawn from knowing that there is heteroscedasticity in a scattergram.
b. Incorrect This is not a conclusion that can be drawn from knowing that there is heteroscedasticity in a scattergram.
c. CORRECT Heteroscedasticity is occurring when variability of scores on the criterion differs for different scores on the predictor - e.g., when there is a narrow range of criterion scores for low and high predictor scores but a wide range of criterion scores for moderate predictor scores. A figure illustrating heteroscedasticity and homoscedasticity is included in the Statistics and Research Design chapter of the written study materials.
d. Incorrect This is not a conclusion that can be drawn from knowing that there is heteroscedasticity in a scattergram.

The correct answer is: the variability (range) of scores on the criterion varies for different scores on the predictor.

205
Q

Parametric statistical tests are usually preferable to non-parametric tests because they are more “powerful.” This means that the use of a parametric test to analyze the data collected in a research study helps ensure that:
Select one:

A.
a false null hypothesis will be retained.

B.
a true null hypothesis will be retained.

C.
a false null hypothesis will be rejected.

D.
the null hypothesis will be rejected.

A

Statistical power is the ability to detect a false null hypothesis.

a. Incorrect This answer describes a Type II error.
b. Incorrect This is not a description of power.
c. CORRECT This is an accurate description of power, which refers to the ability to reject a false null hypotheses. Note: If you’re having trouble remembering what power is or what the difference between a Type I and a Type II error is, the Decision Outcomes table in the Statistics and Research Design chapter may be helpful.
d. Incorrect Power refers to the ability to reject a false null hypothesis, not just to the ability to reject any null hypothesis. Therefore, answer c is the best response.

The correct answer is: a false null hypothesis will be rejected.

206
Q

Which of the following techniques would be most useful for obtaining information on the cognitive processes used by students while they solve complex problems?
Select one:

A.
event sampling

B.
situation sampling

C.
functional analysis

D.
protocol analysis

A

Of the techniques listed in the answers, only one is used to study cognitive processes.

a. Incorrect Event sampling is a behavioral observation technique that is useful when the behavior of interest occurs infrequently.
b. Incorrect Situation sampling is a behavioral observation technique that involves observing a behavior in a variety of situations.
c. Incorrect A functional analysis is used to identify the functions of a behavior - i.e., the antecedents and consequences that maintain the behavior.
d. CORRECT Protocol analysis (also known as the “think aloud” technique) is used to identify the mental processes that occur during the solution of problems or the performance of tasks.

The correct answer is: protocol analysis

207
Q

Of 100 students surveyed on a college campus, 48 voted Republican and 52 voted Democratic in the last election. On another college campus, 63 of the 100 students surveyed voted Republican and 37 voted Democratic. Which of the following statistical tests is the appropriate technique for determining if the difference in voting preferences at the two colleges is significant?
Select one:

A.
two-way ANOVA

B.
students t-test

C.
chi-square test

D.
Mann-Whitney U test

A

In this situation, the analysis will involve comparing the frequencies (numbers) of Republican and Democrat voters at the two schools.

a. Incorrect The two-way ANOVA is used when a study involves two independent variables and data on the dependent variable represent an interval or ratio scale.
b. Incorrect The t-test is used to compare two means.
c. CORRECT The chi-square test is used when data are reported as the frequency of observations, which is the case in this study - i.e., the analysis will involve comparing the number of Democrats versus Republicans at the two colleges.
d. Incorrect The Mann-Whitney U test is used when the analysis involves comparing rank-ordered data obtained from two independent samples.

The correct answer is: chi-square test

208
Q

When the relationship between the predictor (the X variable) and the criterion (the Y variable) is curvilinear and both variables are measured on an interval or ratio scale, the appropriate correlation coefficient is:
Select one:

A.
phi.

B.
tau.

C.
rho.

D.
eta.

A

The choice of a correlation coefficient is based on several factors including the scale of measurement of the variables and the shape of the relationship between them (linear vs. non-linear).

a. Incorrect The phi coefficient is appropriate when both variables are true (natural) dichotomies.
b. Incorrect Kendall’s tau is appropriate when both variables are measured on an ordinal scale.
c. Incorrect Rho (also known as the Spearman rank-order correlation coefficient) is used when both variables are measured as ranks.
d. CORRECT Eta is used to measure the relationship between two continuous (interval or ratio) variables when their relationship is nonlinear.

The correct answer is: eta.

209
Q

The magnitude or strength of the standard error of the mean increases as:
Select one:

A.
the sample size increases and the population variance decreases.

B.
the sample size increases and the population variance increases.

C.
the sample size decreases and the population variance decreases.

D.
the sample size decreases and the population variance increases.

A

Knowing the formula for the standard error of the mean would have helped you identify the correct answer to this question.

a. Incorrect See explanation for response d.
b. Incorrect See explanation for response d.
c. Incorrect See explanation for response d.
d. CORRECT The standard error of the mean is equal to the population standard deviation divided by the square root of the sample size. This formula indicates that, as the population standard deviation increases and/or the sample size decreases, the standard error increases in magnitude. A more detailed description of the standard error of the mean is provided in the Statistics and Research Design chapter of the written study mateials.

The correct answer is: the sample size decreases and the population variance increases.

210
Q

Autocorrelation is most likely to be a problem when using which of the following research designs?
Select one:

A.
time-series

B.
factorial

C.
between groups

D.
Solomon four-group

A

Autocorrelation refers to the correlation between measurements of the dependent variable when it is repeatedly administered to the same participants. Autocorrelation is a problem in repeated measures designs because it can artificially inflate the value of the inferential statistic and thereby increase the probability of making a Type I error.

a. CORRECT Of the research designs listed in the answers, repeated measurement of the dependent variable is characteristic only of the time series design, which is a type of repeated measures (within subjects) design.
b. Incorrect Factorial designs include two or more independent variables and do not necessarily involve repeated measurements of the dependent variable.
c. Incorrect See explanation for response a.
d. Incorrect See explanation for response a.

The correct answer is: time-series

211
Q

When using multiple regression, multicollinearity occurs when:
Select one:

A.
the predictors are highly correlated with one another.

B.
the criterion measures are highly correlated with one another.

C.
there is a low correlation between the predictors and the criterion.

D.
there is a high (significant) correlation between the predictors and the criterion.

A

When choosing predictors for a multiple regression equation, the optimal condition is for each predictor to explain unique variability in the criterion. When predictors are correlated, this condition is violated.

a. CORRECT The term multicollinearity is used to describe high correlations between predictors (X variables). Additional information on multiple regression and multicollinearity is presented in the Statistics and Research Design chapter of the written study materials.
b. Incorrect See explanation for response a.
c. Incorrect See explanation for response a.
d. Incorrect See explanation for response a.

The correct answer is: the predictors are highly correlated with one another.

212
Q

You would use the Solomon four-group design in order to:
Select one:

A.
eliminate carryover effects.

B.
reduce demand characteristics.

C.
evaluate the impact of pretesting.

D.
evaluate the effects of history and maturation.

A

The Solomon four-group design combines the pretest-posttest control group design with the posttest only control group design.

a Incorrect See explanation for response c.

b. Incorrect See explanation for response c.
c. CORRECT The purpose of the Solomon four-group design is to evaluate the impact of pretesting on a study’s internal and external validity.
d. Incorrect See explanation for response c.

The correct answer is: evaluate the impact of pretesting.

213
Q

The “hallmark” of true experimental research is that, in contrast to quasi-experimental research, it permits the researcher to:
Select one:

A.
operationally define the independent and dependent variables.

B.
use parametric tests to analyze the obtained data.

C.
randomly select participants from the population.

D.
randomly assign participants to treatment groups.

A

True experimental research provides the researcher with greater experimental control than does quasi-experimental research.

a. Incorrect An operational definition is a precise description of a variable in a way that facilitates its measurement. Variables are operationally defined when using either a true or quasi-experimental research strategy.
b. Incorrect The choice of a parametric or nonparametric statistical test is based on the characteristics of the data, not the research strategy used.
c. Incorrect Random selection is not a requirement for either true or quasi-experimental research.
d. CORRECT The key feature that distinguishes true and quasi-experimental research is the ability to randomly assign participants to groups in true experimental research, which makes it possible to draw causal inferences about the relationships between variables.

The correct answer is: randomly assign participants to treatment groups.

214
Q

A distribution of 250 scores is positively skewed and has a range of 1 to 50. Which of the following accurately describes the relationship of the measures of central tendency in this distribution?
Select one:

A.
The median is the highest score and the mean is the lowest score.

B.
The mode is the highest score and the mean is the lowest score.

C.
The mean is the highest score and the mode is the lowest score.

D.
The mean is the highest score and the median is the lowest score.

A

In a positively skewed distribution, the “tail” is extended on the high side of the distribution - i.e., most of the scores (outliers) are “piled up” in the low end of the distribution, while a few scores are in its high end.

a. Incorrect See explanation for response c.
b. Incorrect See explanation for response c.
c. CORRECT In a positively skewed distribution, the mean has the highest value; the mode has the lowest value; and the median is between the mean and the mode. A figure illustrating the relationship between the mean, median, and mode in skewed distributions is included in the Statistics and Research Design chapter of the written study materials.
d. Incorrect See explanation for response c.

The correct answer is: The mean is the highest score and the mode is the lowest score.

215
Q

When using stepwise multiple regression to explain the greatest amount of variation in Y using the fewest possible X variables, you:
Select one:

A.
use the Spearman-Brown formula to adjust the size of the multiple correlation coefficient as each X variable is added or subtracted

B.
use R-squared (coefficient of multiple determination) to determine if predictability is substantially affected by adding or subtracting another X variable.

C.
use r (zero order correlation coefficient) to determine which X variable has the highest correlation with the X variables already introduced into the equation.

D.
use the eigenvalues to determine which predictor explains the greatest proportion of the remaining variability in Y.

A

This is a difficult question because the answers are all somewhat convoluted. However, if you recall that multiple regression is used for the purpose of prediction, this would help you pick the correct response.

a. Incorrect The Spearman-Brown formula is used to estimate the effects of increasing or decreasing the length of a test on its reliability coefficient and is not used in this situation.
b. CORRECT The coefficient of multiple determination (which is the square of R) is used to determine if adding or subtracting a predictor has a significant effect on the proportion of variability in Y that has been explained by the predictors included in the multiple regression equation.
c. Incorrect Although it is important to check the correlations between the predictors in order to avoid the problem of multicollinearity, this is not the best response.
d. Incorrect You should have the term “eigenvalues” associated with principal components analysis, not multiple regression.

The correct answer is: use R-squared (coefficient of multiple determination) to determine if predictability is substantially affected by adding or subtracting another X variable.

216
Q

An achievement test is administered to 250 high school seniors. The test’s mean is 50, its standard deviation is 5, and the test scores are normally distributed. If you want to use test scores to select the top 16% of the students, you will set the cutoff score at which of the following?
Select one:

A.
55

B.
60

C.
65

D.
70

A

By the time you take the licensing exam, you’ll want to have memorized the areas under the normal curve so that you can answer questions like this one. A figure that you may find helpful for this purpose is included in the Test Construction chapter of the written study materials.

a. CORRECT In a normal distribution, 50% of examinees obtain scores at the mean or below and an additional 34% obtain scores falling between the mean and the score that is one standard deviation above the mean: 50% plus 34% equals 84%. Consequently, the score that is one standard deviation above the mean divides the distribution into two sections: 84% of the people in the distribution obtain scores below that score and the remaining 16% obtain higher scores. The test described in this question has a mean of 50 and a standard deviation of 5, so the score that is one standard deviation above the mean is 55.
b. Incorrect See explanation for response a.
c. Incorrect See explanation for response a.
d. Incorrect See explanation for response a.

The correct answer is: 55

217
Q

When a test’s reliability has been estimated by splitting the test in half and correlating scores on the two halves, you would use which of the following to correct the resulting reliability coefficient?
Select one:

A.
correction for attenuation formula

B.
standard error of measurement

C.
Kuder-Richardson 20

D.
Spearman-Brown formula

A

Split-half reliability tends to underestimate a measure’s reliability, so a correction formula is often used to estimate what the measure’s reliability coefficient would be if it were based on the full length of the test.

a. Incorrect The correction for attenuation formula is used to estimate the effects of increasing a predictor’s and/or criterion’s reliability coefficient on the predictor”s validity coefficient.
b. Incorrect The standard error of measurement is used to calculate the range within which an examinee”s true score is likely to fall given his/her obtained score.
c. Incorrect Kuder-Richardson 20 (KR-20) is a type of internal consistency reliability that is used when test items are scored dichotomously.
d. CORRECT The Spearman-Brown formula is commonly used to correct the split-half reliability coefficient but can also be used whenever the goal is to estimate the effects of lengthening or shortening a test on its reliability coefficient.

The correct answer is: Spearman-Brown formula

218
Q

Which of the following would be useful for determining the extent to which an examinee’s actual criterion score is likely to deviate from his/her predicted criterion score?
Select one:

A.
standard error of estimate

B.
standard error of measurement

C.
standard error of the difference

D.
standard error of the mean

A

Because the relationship between a predictor and a criterion is never “perfect,” any predicted criterion score for an individual may or may not be his/her actual criterion score. Consequently, a good strategy is to interpret an examinee’s obtained score in terms of a confidence interval.

a. CORRECT The standard error of estimate is used to construct the range within which an examinee’s true criterion score is likely to fall given his/her predicted criterion score. This range is referred to as a confidence interval.
b. Incorrect The standard error of measurement is used to construct a confidence interval around an examinee’s obtained (not predicted) score.
c. Incorrect The standard error of difference is used to construct a confidence interval around a difference score; i.e., a score that is calculated by subtracting scores on two tests (for example, by subtracting Performance IQ from Verbal IQ).
d. Incorrect The standard error of the mean is the standard deviation of the sampling distribution of means and is not used to construct a confidence interval around a predicted score.

The correct answer is: standard error of estimate

219
Q

An educational psychologist designs a screening test to identify underachieving first- and second-grade children who have a learning disability. The psychologist will probably be most concerned that her test has adequate __________ validity.
Select one:

A.
content

B.
construct

C.
concurrent

D.
predictive

A

When a test developer wants to use a screening test to estimate or evaluate current behavior, he/she will be most interested in the test”s concurrent validity.

a. Incorrect Content validity is of most concern when test items are expected to be a representative sample of a particular content domain.
b. Incorrect Construct validity is of concern for tests designed to measure a hypothetical trait or construct (e.g., motivation, self-esteem).
c. CORRECT As noted above, concurrent validity (a type of criterion-related validity) is of concern when the purpose of a test is to estimate an individual”s current status on some external criterion. Screening tests are usually used to estimate the results of a more thorough evaluation (the criterion) which, in this case, would be a more thorough diagnostic procedure.
d. Incorrect The screening test is being used to estimate current status, not to predict future status. Therefore, concurrent validity would be of greater concern than would predictive validity.

The correct answer is: concurrent

220
Q

An assumption underlying _____________ is that observed covariation among a set of variables is due to one or more underlying common dimensions.
Select one:

A.
factor analysis

B.
path analysis

C.
trend analysis

D.
multiple regression

A

Covariation is another word for correlation, so this question is asking what technique is used to determine what accounts for the intercorrelations among a set of tests or other variables. You may have been able to answer this question using the process of elimination if you’re familiar with the techniques listed in the incorrect answers.

a. CORRECT The primary function of factor analysis is to identify the factors (dimensions) that explain or underlie the intercorrelations among a set of tests or other variables.
b. Incorrect Path analysis is used to test causal models for the relationships among a set of variables.
c. Incorrect Trend analysis is a type of analysis of variance that is used when the independent variable is quantitative.
d. Incorrect Multiple regression is used when the goal is to predict a score on a criterion based on scores on two or more predictors.

The correct answer is: factor analysis

221
Q

An item discrimination index (D) of ____ indicates that all examinees in the high-scoring group and none in the low-scoring group answered the item correctly.
Select one:

A.
1

B.
-1

C.
0.5

D.
0.0

A

The item discrimination index ranges from -1.0 to +1.0 and is calculated by subtracting the percent of low-scoring examinees who answered the item correctly (examinees who obtained low total scores on the test) from the percent of high-scoring examinees who answered the item correctly (examinees who obtain high total scores on the test).

a. CORRECT An item discrimination index of +1.0 indicates that all of the high-scoring and none of the low-scoring examinees answered the item correctly.
b. Incorrect An item discrimination index of -1.0 indicates that all of the low-scoring and none of the high-scoring examinees answered the item correctly.
c. Incorrect An item discrimination index of .50 indicates that the 50% fewer of the examinees in the low-scoring group (versus the high-scoring group) answered the item correctly.
d. Incorrect An item discrimination index of 0 indicates that the same percent of examinees in the low- and high-scoring groups answered the item correctly.

The correct answer is: 1

222
Q

When a predictor’s reliability coefficient is .81, its criterion-related validity coefficient can be:
Select one:

A.
no less than .81.

B.
no greater than .81.

C.
no less than .90.

D.
no greater than .90.

A

A test’s reliability limits its validity – i.e., an unreliable test cannot be valid. The formula for the relationship between reliability and validity is given in the Test Construction chapter of the written study materials, and knowing the formula would have helped you identify the correct answer to this question.

a. Incorrect See explanation for response d.
b. Incorrect See explanation for response d.
c. Incorrect See explanation for response d.
d. CORRECT A predictor’s validity coefficient can be no greater than the square root of its reliability coefficient. In this situation, the predictor’s reliability coefficient is .81, so its validity coefficient can be no greater than the square root of .81, which is .90.

The correct answer is: no greater than .90.

223
Q

In test construction, shrinkage is associated with:
Select one:

A.
differential validity.

B.
incremental validity.

C.
cross-validation.

D.
process validation.

A

In the context of test construction, shrinkage refers to a reduction in the magnitude of a measure’s criterion-related validity coefficient.

a. Incorrect A test has differential validity when it has different validity coefficients for different groups.
b. Incorrect Incremental validity refers to the increase in decision-making accuracy that results from use of a predictor.
c. CORRECT Cross-validation refers to validating a predictor with a new sample. Because the predictor is often “tailor-made” for the original validation sample (i.e., items are included in the predictor based on their correlation with the criterion for that sample), the chance factors that contributed to the original validity coefficient may not be present in the cross-validation sample. As a result, the validity coefficient for the cross-validation sample is usually smaller.
d. Incorrect Process validation refers to determining that a process will continue to meet pre-determined standards and is not relevant to the phenomenon known as shrinkage.

The correct answer is: cross-validation.

224
Q
The slope (steepness) of an item response curve indicates which of the following?
Select one:

A.
the items ability to discriminate between high and low achievers.

B.
the items difficulty level

C.
the susceptibility of the item to social desirability effects

D.
the probability that the item can be answered correctly by guessing

A

The various item response theory models produce item response curves (ICCs) that provide information on either one, two, or three parameters - difficulty level, discrimination, and probability of guessing correctly.

a. CORRECT An item’s ability to discriminate between those who have high and low levels of the attribute measured by the item is indicated by the slope of the ICC - the steeper the slope, the greater its discrimination.
b. Incorrect An item’s difficulty level is indicated by the ability level (charted on the horizontal axis) at which 50% of examines in the sample answered the item correctly.
c. Incorrect Susceptibility to social desirability is not indicated by the ICC.
d. Incorrect The probability of guessing correctly is indicated by the point at which the ICC intercepts the vertical axis.

The correct answer is: the items ability to discriminate between high and low achievers.

225
Q

When assessing the test-retest reliability of your newly developed personality test, you obtain a reliability coefficient of .80. This means that:
Select one:

A.
80% of variability in test scores is shared variability.

B.
64% of variability in test scores is shared variability.

C.
80% of variability in test scores is true score variability.

D.
64% of variability in test scores is true score variability.

A

A reliability coefficient is interpreted directly as a measure of true score variability.

a. Incorrect See explanation for response c.
b. Incorrect See explanation for response c.
c. CORRECT Reliability is a measure of the extent to which a test consistently measures the attribute(s) assessed by the test - i.e., the extent to which scores reflect true score variability. Note that the reliability coefficient is not squared in order to interpret it but is, instead, interpreted directly.
d. Incorrect See explanation for response c.

The correct answer is: 80% of variability in test scores is true score variability.